Вы находитесь на странице: 1из 52

Wills and Trusts

Law School Legends

Professor Stanley M. Johanson


I. INTESTATE SUCCESSION
A.

BASIC PRINCIPLES AND TERMINOLOGY.


Probate (literal translation: to prove): Court proceeding in which:
(i)

it is judicially determined that the decedent left a validly executed will (or that the
decedent died without a will and his intestate heirs are determined),

(ii)

a personal representative (called an executor if named in a will, an administrator if


appointed by the court from a statutory list of preferred next of kin) is appointed to
administer the decedent's estate and wind up the decedents affairs, and

(iii) letters testamentary (if an executor) or letters of administration (if an administrator)


are issued by the court, showing the personal representatives authority to represent the
estate in dealing with third parties. The probate estate does not include (and wills and
the intestate succession rules do not apply to) non-probate transfersinterests that
pass by right of survivorship (e.g., a joint and survivor bank account) or under the
terms of a contract (e.g., life insurance proceeds or employee death benefits).
Strictly speaking, you probate the decedents will and then you administer the decedents estate.
However, the entire process is commonly referred to as the probate process, involving a probate
administration. While it may not be technically correct, this terminology is widely used.
The principal duties of a personal representative are to:
(i)

take possession and control of the assets that comprise the estate,

(ii)

give notice to creditors and pay creditors claims,

(iii) satisfy the tax authorities, and


(iv)

distribute the remaining estate to the will beneficiaries or heirs.

A person who executes a will is called the testator. Persons who take by intestate succession are
called heirs. Persons who take under a will are called beneficiaries (or devisees, or legatees). At
common law, one devised real property and bequeathed money or personal property. This
distinction is no longer made. A will can say I devise, I bequeath, I give, or I leaveall of
these terms mean the same thing, and can be employed to make testamentary gifts of both real
property and personal property.

Page 2 WILLS AND TRUSTS

LAW SCHOOL LEGENDS

Residuary estate is the estate of a testator that remains after all debts, taxes and administration
expenses have been paid, and after all specific gifts and cash legacies made by the will have been
satisfied. Under most wills, the residuary beneficiary is the testators preferred taker. (E.g., "I give
all of the rest, residue and remainder of my estate to my wife Wendy.")
B.

WHEN INTESTATE DISTRIBUTION RULES APPLY.


The intestate distribution rules (sometimes referred to as the laws of descent) apply when:
(i)

decedent left no will (or decedents will was not validly executed);

(ii)

the will does not make a complete disposition of the estate (resulting in a partial
intestacy), or

(iii) an heir successfully contests the will on the ground of lack of testamentary capacity or
undue influence, and the will is denied probate. In several states, the intestacy rules are
also involved in questions involving a pretermitted child (born or adopted after the will
was executed) or an omitted spouse (testator married after will was executed.)
C.

INTESTATE SHARE OF SURVIVING SPOUSE


Uniform Probate Code (original 1969 version), which has been enacted in a number of states, has
a different rule depending on whether the decedent's descendants were of this marriage or of an
earlier marriage. Under the original UPC, the surviving spouse's intestate share is:
Survived by spouse but not by descendants or parents: ALL.
Survived by descendants, all of whom are descendants of surviving spouse:
$50,000 PLUS OF ANY BALANCE.

THE FIRST

Survived by descendants, some of whom are not descendants of surviving spousethe second
marriage situation: ONE-HALF.
Survived by spouse and no descendants but by one or both parents:
PLUS OF ANY BALANCE.

THE FIRST $50,000

Revised Uniform Probate Code (1990 version) substantially increased the intestate share of a
surviving spouse, reflecting a policy that is more in line with the dispositions that most spouses
make if they write wills. The Revised UPC provisions also reflect that divided families are
encountered more frequently in our society.
Survived by spouse but not by descendants or parents: ALL.
Not survived by descendants, but survived by a parent: THE FIRST $200,000 PLUS OF
ANY BALANCE.
Survived by descendants, all of whom are descendants of surviving spouse, and surviving
spouse has no descendants from an earlier marriagethe one-marriage situation: ALL.
Survived by descendants, all of whom are descendants of surviving spouse, but surviving
spouse has one or more descendants who are not descendants of the decedent: THE FIRST
$150,000 PLUS OF ANY BALANCE.
Survived by descendants, some of whom are not descendants of surviving spouse: THE
FIRST $100,000 PLUS OF ANY BALANCE.

LAW SCHOOL LEGENDS

WILLS AND TRUSTS Page 3

1. Herman and Winnie are married, and they have two children (Al and Betty). Winnie has a
child Carol) by her first marriage.

H-1

Winnie

Carol

Al

Herman

Betty

a. If Herman dies intestate survived by Winnie and the three children, what is Winnies
intestate share:

Under the original (1969) Uniform Probate Code?

Under the Revised (1990) Uniform Probate Code?

b. If Winnie dies intestate survived by Herman and the three children, what is Hermans
intestate share:

Under the original (1969) Uniform Probate Code?

Under the Revised (1990) Uniform Probate Code?

D.

INHERITANCE BY DESCENDANTS.
The Probate Codes of a few states use the term "issue" throughout. The terms issue and
descendants are synonymous, and include lineal descendants (children, grandchildren, etc.) by
blood or adoption. In making a distribution among descendants, the literal translation of per stirpes
is by the roots, and means one share for each line of descendants. The literal translation of per
capita is by the head, and means one share for each person.

Page 4 WILLS AND TRUSTS

LAW SCHOOL LEGENDS

2. Martha, a widow, dies intestate, survived by the family members listed below. What intestate
distribution?
Martha
Al

Ben

Carol

C-1

Donna

C-2

D-1

D-2

D-3

At common law, and in a small number of states today, the distribution rule is strict per
stirpes (also called classic per stirpes), under which the shares for each line of descendants
are always divided at the first generational level, regardless of whether there are any living
takers at that level, and then one share for each family line. Thus in the above family tree:
Al: ________

Ben: _______

C-1 and C-2: ________

D's three children: ________

The distribution rule under the original UPC and in most states is modern per stirpes,
usually described as per capita with representation ("per capita at the first level, then by
representation"). Under this distribution scheme, you cut the shares at the first generational
level at which there are living takers, and then one share for each family line.
Al: ________

Ben: _______

C-1 and C-2: ________

D's three children: ________

The distribution rule under the Revised UPC (also adopted in several non-UPC states) is per
capita at each generation. Under this rule, you make the initial division of shares (with one
share for each line of descendants) at the first generational level at which there are living
takers. Each living descendant in that nearest generation takes one share. Shares of deceased
persons at that generational level are combined and then divided equally among the takers at
the next generational level, and so on. As a result, persons in the same degree of kinship to
the decedent (e.g., grandchildren who are first cousins) always take equal shares. Under this
distribution rule, the shares would be:
Al: _________

E.

Ben: _________

C-1 and C-2: __________

D's children: __________

INTESTATE DECEDENT NOT SURVIVED BY SPOUSE OR DESCENDANTS


(1)

All to parents or to surviving parent (majority rule). In nearly all states, collateral kin
(brothers and sisters) never inherit if an intestate decedent is survived by a parent.)

(2)

If not survived by parents, to descendants of parents. When inheritance is by collateral kin


(e.g., brothers, sisters, and the descendants of deceased brothers and sisters), the states apply
the same distribution rules (either strict per stirpes, or per capita with representation, or per
capita at each generation) that they apply to inheritance by descendants.

LAW SCHOOL LEGENDS

F.

WILLS AND TRUSTS Page 5

(3)

The UPC and most states have abolished the distinction made at common law between
collateral kin of the whole blood and of the half blood: Half-sisters (sisters who shared one
common parent with the decedent) take the same share as sisters of the whole blood.

(4)

If not survived by parents or issue of parents: to maternal grandparents (or surviving


grandparent), or if neither maternal grandparent is living, their descendants; to paternal
grandparents (etc. etc.) in same manner. If no maternal grandparents or their descendants, all
to paternal grandparents or their descendants (and vice versa).

(5)

While most states impose no limit on the degree of kinship needed to take as an heir, the
Uniform Probate Code and several non-UPC states have enacted so-called "no laughing
heir" statutes: There is no inheritance by kin more remotely related than grandparents or
descendants of grandparents. Instead, the estate escheats to the state.

ADOPTED CHILDREN. Adopted children and their descendants have full inheritance rights
from the adoptive family (and vice versa), and are treated in all respects the same as natural
children.
General rule: Once a child has been adopted by a new family, the child has no inheritance rights
from the natural parents or their kin. (This is consistent with the law and policy of most states,
under which adoption records are sealed, and an adopted child has no right to learn the identity of her
natural parents.)
Exception: Where child is adopted by spouse of a natural parent. (E.g., Clyde's father dies; mother
remarries, and second husband adopts Clyde. Clyde has inheritance rights from natural mother and
adoptive fatherand (in nearly all states) from the deceased natural fathers kin as well.)

G.

NONMARITAL CHILDREN. Constitutional litigation in 1970s and 1980s expanded the rights
of nonmarital children. As a result, in most states a nonmarital child can inherit from natural father
if [the acronym is PAPfor Pappy]:
-- Paternity suit: The man was adjudicated to be the father in a paternity suit; or
-- Acknowledged paternity: The man acknowledged in writing that he was the father; or
-- Probate proceedings: After the man's death, he is proved to have been the father of the child
in the probate proceeding by clear and convincing evidence.

H.

LIFETIME GIFTS TO HEIR OR WILL BENEFICIARY


3. Mary gives Blackacre (worth $60,000) to her son Al on Al's 35th birthday, orally telling her
other sons (Ben and Chris) that they will receive similar gifts when they reach 35. Mary dies
intestate two years later without having made gifts to Bill and Chris. She left an estate valued
at $300,000; what distribution?
Common law: A lifetime gift to a child (or other descendant) was presumptively an
advancement (advance payment) of the child's intestate share, to be taken into account in
distributing the intestate estate at death. (Presumption was that a parent would want to
treat all children equally.) Thus if Al wanted to share in the inheritance, unless the
advancement presumption were rebutted it would be treated as a [$300,000+ $60,000 =]
$360,000 estate to be divided three ways. Each child's share would be $120,000, with Al

Page 6 WILLS AND TRUSTS

LAW SCHOOL LEGENDS

already having received $60,000 of his share. (In most states, the advanced property is
valued at its date-of-gift value for this purpose.)
UPC and majority rule: A lifetime gift to an heir is not an advancement unless
(i) declared as such in a _______________________________________ by donor; or
(ii) acknowledged as such in a ____________________________________ by donee.
Therefore, under the majority rule distribute Marys estate:

In most states, the same rules apply to lifetime gifts made to a beneficiary named in a
previously executed will (called the doctrine of satisfaction of legacies), as where a
testator executes a will bequeathing $25,000 to her nephew Norman, and thereafter
gives Norman $10,000 cash.
UPC and majority rule: A gift to a beneficiary named in an earlier will is not treated
as in satisfaction of legacy unless
(i) declared as such in a contemporaneous writing by the donor, or
(ii) acknowledged as such in writing by the donee, or
(iii) the will provides for reduction of legacies by any lifetime gifts.
Since there is no writing, we ignore the lifetime gift. Norman takes the full $25,000
under the will.

I.

DISCLAIMER BY HEIR OR BENEFICIARY


4. Joe's will bequeaths his $1,500,000 estate "to my children in equal shares." Joe is survived
by two children: Sue and Bob. Sue, a partner in a large New York law firm who has two
children, wants to disclaim her interest in her father's estate. What must Sue do to make an
effective disclaimer?
All states recognize that no one can be compelled to be a beneficiary or heir against her will.
An intestate heir, will beneficiary, beneficiary of a life insurance policy or employee benefit
plan, or any other interest in property can disclaim the interest, in whole or in part. A
disclaimer also can be made on behalf of a minor or incapacitated person by a guardian, or
the personal representative of a deceased person. A disclaimer, once made, is irrevocable.
To be a valid disclaimer:
#1. Must be in writing, signed and (in most states) notarized.
#2. Must be filed within _________________* after the decedent's death.
* Under the Uniform Probate Code, a disclaimer must be made within a
reasonable time. However, to be valid for tax purposes a disclaimer must be
made within 9 months, and most disclaimers are made for tax purposes.

LAW SCHOOL LEGENDS

WILLS AND TRUSTS Page 7

[For irrevocable inter vivos trusts, disclaimer must be made within 9 months
of the transfer; the trust is read as through the disclaimant was dead when trust
was created.
#3. Disclaimer can be partial ("I disclaim one-half of the interest bequeathed to me").
#4. A beneficiary or heir cannot disclaim after accepting an interest or its benefits.
(Essentially an estoppel principle.)
#5. Disclaimant cannot exercise dominion by attempting to direct who takes by reason of
the disclaimer. ("I disclaim, and I want the interest to go to my husband Horace.")
If Sue makes an effective disclaimer, how should T's estate be distributed?

J.

DEATHS IN QUICK SUCCESSION


5. Mother and Sonny, riding in a car, are hit broadside by a train. Both die instantly. Mother
did not leave a will. For purposes of distributing Mother's intestate estate, is Sonny an heir....
________ If the controlling law is the Uniform Simultaneous Death Act [USDA]?
Uniform Simultaneous Death Act: When title to property depends on order of deaths
and there is no sufficient evidence that the persons have died otherwise than
simultaneously, the property of each passes as though he or she survived (absent contrary
provision).
Intestacy: As though the intestate survived and the heir predeceased.
Wills: Suppose, instead, that Mother left a will devising "all my property" to Sonny.
Mother's estate is distributed as though the testator survived and the beneficiary
predeceased. (This would invoke the lapsed gift doctrine and the anti-lapse statute,
discussed infra.)
Insurance proceeds, Individual Retirement Accounts [IRAs], etc.: As though insured [or
IRA account owner] survived and beneficiary predeceased.
If joint tenants with right of survivorship or tenants by the entirety die simultaneously:
is distributed through As estate as though A survived B, and is distributed through
Bs estate as though B survived A. Reason: Simultaneous deaths prevent operation of right
of survivorship. In effect, property passes as though a tenancy in common were involved.
5a. Same facts and same tragic accident except that, while Mother was pronounced dead at the
scene of the accident, Sonny died at a nearby hospital 71 minutes later. Mother did not leave
a will. For purposes of distributing mother's intestate estate, is Sonny an heir....
________ If the controlling law is the Uniform Simultaneous Death Act?

Page 8 WILLS AND TRUSTS

LAW SCHOOL LEGENDS

________ If the state has enacted the Uniform Probate Codes 120-hour rule?

The 120-hour rule also applies to wills. A will beneficiary who fails survive the testator by
120 hours is deemed to have predeceased the testator (absent contrary provision). This
would invoke that lapsed gift rule and anti-lapse statute (discussed infra).

II. EXECUTION OF WILLS


A.

REQUIREMENTS FOR A VALIDLY EXECUTED WILL.


To have capacity to make a will, the testator (T) must be 18 years old. The rules governing will
execution vary from state to state. The Uniform Probate Code (UPC) requires:
#1.

Signed by the testator (or someone at T's direction and in her conscious presence
proxy signature);

#2.

Testator must sign the will (or acknowledge his earlier signature or acknowledge the
will as his will) in each witnesss presence;

#3.

Two attesting witnesses, each of whom witnessed either Ts signing of the will or T's
acknowledgment of his earlier signature or of the will; and

#4.

Witnesses must sign within a reasonable time after witnessing either T's signing of the
will, or Ts acknowledgment of her earlier signature or of the will.

A number of states additionally require that


#5.

Each witness must sign the will in the testators presence.


notwithstanding, this is the majority rule.)

(The UPC

A few states impose one or more of the following additional requirements:


--

that the testator sign "at the foot or end" of the will.

--

that the witnesses know they are witnessing a will, as distinguished from some other
document (called the "will publication" requirement).

--

that witnesses sign in each other's presence.

Codicil (later amendment or supplement to a will) must be executed with the same formalities.
6. Tom types a will that leaves all of all his property to his sister Sue and his neighbor Nell in
equal shares. The will, which names a friend as executor, contains no attestation clause;
below the signature line for the testator the will simply provides "Witnesses" and has two
signature lines. Tom takes the will across the street to his neighbor Nell and asks Nell to
"witness my will." Nell signs on the first witness line, then Tom signs; Tom's signature is
barely legible because of an arthritic condition. Tom then takes the will to his neighbor
Oscar and asks: "Would you mind witnessing this legal document for me? It needs two
signatures besides mine." Tom proffers the will with his signature showing. Oscar signs,
thinking he is witnessing a power of attorney. After Toms death the will, which is undated,
is offered for probate. Should it be admitted?

LAW SCHOOL LEGENDS

WILLS AND TRUSTS Page 9

_______ Does it matter that the will is not dated?


_______ Does it matter that one of the witnesses signed before Tom signed?

The exact order of signing is not critical when execution (will signing) ceremony is
_________________________________________________________________
(But where T forgot to sign when witnesses signed, and added his signature three days later
in the same witnesses' presence, will denied probate; not a contemporaneous transaction.
Witnesses are attesting witnesses and must attest to T's signature when they sign.)
_______ Does it matter that Tom's signature is barely legible?

_______ Does the fact that Nell is a beneficiary invalidate either the will or the bequest
to Nell?
Minority rule: Interested witness situation doesn't affect validity of a will, but
witness-beneficiary loses legacy (subject to exceptions). However, the UPC and most
states have abolished the interested witness rule. "The signing of a will by an
interested person does not invalidate the will or any provision of it."
________ Does it matter that Oscar did not know that he was witnessing a will?
(majority rule)

________ Does it matter that Nell and Oscar didn't sign in each other's presence?
(majority rule)

________ Does it matter that Toms will did not contain an attestation clause?

Attestation clause, which appears below the testator's signature line and above the witnesses'
signature line, recites the elements required for due execution: "On the above date testator declared
to us that the foregoing instrument was her will and she asked us to serve as witnesses thereto. She
then signed the will in our presence, we being present at the same time. We now sign the will as
attesting witnesses in testator's presence and in the presence of each other."

Page 10 WILLS AND TRUSTS

LAW SCHOOL LEGENDS

Value of an attestation clause: such a clause is:

An attestation clause can be important in two situations:


(1)

Witness with bad memory. "Probate of a will does not turn on memory of the attesting
witnesses."

(2)

Hostile witness.

Self-proving affidavit procedure now available in nearly all states recognizes that most probates
are harmonious, nonlitigious affairs in which no one is challenging the validity of the will's
execution. Can be executed at any time after the will is signed, but invariably is signed at the same
ceremony. T and witnesses sign will, then T and witnesses execute sworn affidavit before notary
public. Affidavit recites statements witnesses would testify to in open court (T was over age 18;
witnesses signed in T's presence; in witnesses' opinion T was of sound mind, etc.). Unlike an
attestation clause (which merely corroborates the witnesses' testimony), the affidavit serves the
same function as a deposition or interrogatory. It is a total substitute for live testimony of the
attesting witnesses in open court.
7. Tillie downloads a will form from the Internet, and carefully fills in the blanks by typing in
the names of the beneficiaries, executor, etc. After she has completed the form, Tillie asks
Norman, a notary public, to assist her in signing the will. Tillie signs; then Norman signs and
affixes his notarial seal, overlooking the fact that there are two signature lines for witnesses.
Tillie dies two years later.
NO

Can Tillies will be admitted to probate in most states, given that it has only
one witness?

YES

Can Tillies will be admitted to probate in a state that has enacted the Revised
Uniform Probate Code?

Under the Revised UPCs dispensing power statute (also known as the harmless error
statute), the probate judge can excuse full compliance with the formalities required for
execution of a will if there is clear and convincing evidence that the testator intended the
document to be her will.
B.

WHAT CONSTITUTES PRESENCE? A number of states require that the witnesses must sign
in testator's presence, and the UPC requires that the testator either sign the will or acknowledge his
signature or the will in each witness's presence. What constitutes "presence"?
8. T is confined to a hospital with a contagious disease, his bed hidden by a heavy vinyl screen.
The will is handed to T at a point when the two witnesses are standing in the doorway to the
room, not in T's line of sight because of the screen. From behind the screen T says, "This
looks all right; where do I sign?" After T signs the will, a nurse carries the will to the
doorway where the two witnesses sign under the attestation clause; T cannot see either of
them sign. Was the will validly executed?
Under the line of sight ("scope of vision") test (minority rule): NO Witnesses must be
in testator's line of sight. Testator does not have to see the witnesses sign, but the
witnesses must be within the uninterrupted range of testator's vision when they sign, so
that testator could have seen them sign if he had looked. (Swivel chair case: If testator,

LAW SCHOOL LEGENDS

WILLS AND TRUSTS Page 11

staring out window when witnesses signed on a table behind her, had turned around in her
swivel chair, she could see the witnesses.)
Under the conscious presence test (majority rule): YES It is not necessary that testator
should actually be able to see the witnesses when they sign. They are in his presence
whenever he is so near to them that he is conscious of where they are and what they are
doing, and he could see them with a slight physical effort on his part. (e.g., by peeking
around that vinyl screen.)
BUT where (after testator and W-1 signed) the attorney took the will to an adjoining
room where W-2 signed it, W-2 did not sign in testator's presence under either test.

9. Mel was in a hospital's Intensive Care Unit, having suffered a heart attack. A will was
prepared for Mel and was brought to the hospital room along with two of Mel's neighbors
who were to serve as witnesses to the will. The lawyer read the will aloud to Mel, who said
"That's just fine; where do I sign?" Mel signed the willbut immediately thereafter suffered
a massive seizure and collapsed back on the bed. Paramedics rushed in and, in the presence
of the stunned neighbors, worked alongside two nurses in administering CPR to Melto no
avail. About seven minutes later, Mel was pronounced dead. At that point, the lawyer said to
the neighbors, "well, that's too bad; but as long as you're here you might as well sign the will
as witnesses," which they did. Is Mel's will admissible to probate?
NO

______

C.

Majority rule: Witnesses must sign in the testator's conscious presence.

Uniform Probate Code:

ATTORNEY LIABILITY TO WILL BENEFICIARIES FOR NEGLIGENCE


10. In a state that requires that the attesting witness must sign the will in the testators presence,
Larry Lawyer prepares a will for Tina and supervises the will's execution. The will is signed
by two witnesses, but one of them did not sign the will in Tina's presence. As a result, the
will is denied probate, and Tina's estate passes by intestacy to Tina's heirs. Do the intended
will beneficiaries have a cause of action against attorney for negligence?
Minority rule:

NO

because there is no privity of contract.

Under the minority rule, an attorney's duty runs only to:

Majority rule: _______ because ______________________________________.


Under the majority rule, attorney's duty also runs to:

Page 12 WILLS AND TRUSTS

D.

LAW SCHOOL LEGENDS

HOLOGRAPHIC WILLS
11. Winkie writes a document in her own handwriting that reads: "July 24, 1998. I, Winkie
Waters, declare that this is my last will. I leave all my property to the Morris Crippled
Children's Home." The writing is not witnessed; is it admissible to probate?
"The UPC and about half the states recognize holographic willshandwritten and signed
but unwitnessed wills. In these states, upon proof that the will was wholly in Winkies
handwriting, it is admissible to probate as a holographic will."
But all states that allow holographic wills require that such wills be signed by the testator. Was
this will signed by Winkie???

"However, many states do not recognize holographic wills. Except (in a few states) for
persons serving in the armed forces or mariners at sea, all wills must be in writing and
attested by two witnesses. In those states, the handwritten, unwitnessed document is not
admissible to probate."
Important note: If the facts of ANY Wills question include a handwritten letter, note or
memorandum signed by the testator, you must discuss majority and minority rule (as above);
whether it can be given effect as a holographic will (or holographic codicil to an attested will).

E.

CONDITIONAL WILL
12. Ted writes a will that is properly signed and witnessed: "I am going on a mountain-climbing
expedition to the Himalayas. If anything happens to me on the trip, I leave all of my property
to my good friend, Alice Adams." Ted climbs Mt. Everest that summer, returns from the trip
in July, and dies three years later without having changed his will. Does Alice take Ted's
estate under the will?

Was this a conditional will, meaning that probate should be denied because the condition did
not occur (i.e., nothing happened to Ted on his trip)?

Or did Ted's reference to the dangerous journey he was about to undertake merely reflect the
motive or inducement for making a will? (i.e., the dangers he faced on the trip prompted
him to think of the possibility of death and the need for a will.)

LAW SCHOOL LEGENDS

WILLS AND TRUSTS Page 13

III. REVOCATION OF WILLS


A.

WHAT CONSTITUTES A VALID REVOCATION.


A will can be revoked only:
(1) by a subsequent testamentary instrument, executed with appropriate formalities, or
(2) by physical act (burning, tearing, canceling, obliterating, or other act of destruction).
Under the Revised UPC, the dispensing power (harmless error) statute applicable to the
execution of wills also applies to attempted revocations. The probate court can relax the statutory
requirements, if there is clear and convincing evidence that the testator intended to revoke her will.
13. Hobie's will is found among his papers after his death. At the bottom of each page of the
three-page will is written, in T's handwriting, "This will is VOID. Hobie Gates."
YES

Valid revocation by subsequent instrument if the state recognizes


holographic wills?
It's in his handwriting and signed; doesn't have to be on a separate sheet of
paper.

YES

Valid revocation by subsequent instrument if the state has enacted the


Uniform Probate Codes dispensing power statute?
We have clear and convincing evidence that Hobie intended to revoke his
will.

________ Valid revocation by subsequent testamentary instrument if the state does not
recognize holographic wills (and has not enacted the UPCs dispensing
power statute)?

________ Valid revocation by physical act?

________ Suppose Hobie had crossed out his signature with an X. Revocation by
physical act?

14. The executed copy of Adam's will is in his safety deposit box; a xerographic copy showing
all of the signatures is in Adam's desk at home. Adam destroys the xerox copy with the intent
to revoke his will. Valid revocation by physical act?

________ Physical act must be:

Page 14 WILLS AND TRUSTS

LAW SCHOOL LEGENDS

15. Ted calls his attorney on the phone and tells him, "Revoke my will. I'll come down to your
office next Monday and write a new one, but for now I don't want that old will." The
attorney gets Ted's will out of the file, puts a large X across each page, and then tears the will
into twelve pieces. He tells Ted, "I have destroyed your will. Be sure to come in on Monday.
We sure wouldn't like to have you die intestate. Heh, heh." Ted is struck by lightning and is
killed on Sunday. Was Ted's will validly revoked?
________ Revocation by physical act by another person (proxy revocation), must be:
(1) ______________________________ and (2) ______________________________

15a.

But how can Ted's will in #15 be probated if it was destroyed???


______________________________________________________________________
(1) Proof of due execution (testimony of attesting witnesses) as in any case.
(2) Cause of will's nonproduction must be established.
presumptions as to revocation set out below.)

(Must overcome

(3) Contents must be substantially proved by copy of will or by testimony of


witnesses who have read the will or heard it read.

B.

PRESUMPTIONS REGARDING REVOCATION.


(1) Where a will, last seen in testator's possession or under his control, is not found after death,
presumption is that the testator revoked the will by physical act.
(2) Where a will, last seen in testator's possession or control, is found mutilated after testator's death,
presumption is that the testator did the mutilating (i.e., revocation by physical act).
(i)

Neither presumption arises if the will was last seen in the possession of someone adversely
affected by its contents. [Recent case: Shortly after T's death, desk where will was located
was "tidied up" by one adversely affected by the will.]

(ii)

Evidence is admissible to rebut the presumption of revocation where will cannot be


found or is found in damaged condition. [E.g., will destroyed in fire that killed testator.]

Where a will is executed in duplicate (two signed and witnessed copies), both copies must be
accounted for, or the reason for not producing both copies must be explained.
16. Tim executes "my last will." Two years later, Tim executes another "my last will." The
second will does not contain language of revocation, and does not even mention the earlier
will.
To the extent possible, you read the two instruments together. The second "last will" is
treated as a codicil to the first will, and revokes it only to the extent of inconsistent
provisions. But if the second will is wholly inconsistent with the earlier will (the first will
gives "all my property to Al" and the second gives "all my property to Betty," the first will
is revoked by implication.

LAW SCHOOL LEGENDS

WILLS AND TRUSTS Page 15

Revocation of codicil to a will does not revoke the will, and (majority rule) the part of the
will that was modified or revoked by the codicil is restored and takes effect as though the
codicil had never been written.

C.

DEPENDENT RELATIVE REVOCATION


16. Teresa executes a will (Will #1) that bequeaths her estate in trust: income to daughter Sally
for life, remainder to Sally's children. Teresa later types out a will (Will #2) that purports to
revoke Will #1 and leaves her estate outright to Sally. However, Will #2 is signed by only
one witness. Saying "this will is no good now," Teresa destroys Will #1. Teresa dies; she is
survived by Sally and by her son Jack, whom she intended to disinherit. What do we do,
given that the second "will" is not a will (not validly executed)?
DEPENDENT RELATIVE REVOCATION should be applied by the court. "DRR"
permits a revocation to be disregarded when the act of revocation was premised upon,
conditioned upon, dependent upon, a mistake of law or fact as to the validity of another
disposition (here, Teresa's mistaken belief that she had validly executed Will #2). Effect
would be to disregard the revocation of Will #1 (the will that was revoked based on a
mistake of law) and permit its probate. Rationale: It is better to disregard the revocation
and let Teresa's estate pass into a trust for Sally and her children than to have half of her
estate pass by intestate distribution to Jack, whom Teresa intended to disinherit.
DRR is sometimes called the second best solution doctrine. [The best solutiongiving
effect to Teresa's intent by probating Will #2isnt possible because it was not properly
witnessed.] The doctrine should be applied only if by disregarding the revocation we
come closer to what the testator tried but failed to do than would an intestate distribution.
If Will #1 said "outright to Sally" and the improperly witnessed Will #2 said "outright to
Hobie Gates" and the other facts were the same, DRR would not be applied. T's
revocation of Will #1 (in effect, "I do not want Sally to take") would be independent of
her intent to make a new will in favor of Hobie. To disregard the revocation of Will #1
would defeat T's intent. On these facts, the conventional rules would be applied and an
intestate distribution would be made.
But if we disregard the revocation of Will #1 because the court decides to apply DRR,
how can we probate that will when Teresa has destroyed it???
_______________________________.

D.

CHANGES ON FACE OF WILL AFTER IT HAS BEEN SIGNED AND WITNESSED


18. Elsie's typewritten will makes a number of legacies, including:
"10. I give the sum of $5,000 to my nephew, Hobie Gates."
"11. I give the sum of $2,000 to my niece, Susan Slade.
Elsie decides to make some revisions in her will without the assistance of an attorney. Using
a marking pencil, she deletes clause 10 and strikes the $2,000 in clause 11. Using a ball point
pen, Elsie writes in $5,000 above the crossed out $2,000, and initials and dates the margin
alongside these changes. Elsie dies three years later. What is the effect of these changes on
the face of the will?

Page 16 WILLS AND TRUSTS

LAW SCHOOL LEGENDS

________ Was the legacy to Hobie Gates validly revoked? (Majority rule)
"A few states (Illinois, New York, Texas) do not recognize partial revocation by
physical act. However, in most states and under the Uniform Probate Code, partial
revocations by physical act are valid."

________ Does Susan get the $5,000 legacy? (Overwhelming majority rule)
Words added to a will after it has been signed and witnessed are:

But under the Revised Uniform Probate Code (that harmless error or dispensing power
statute once again), the answer is YES. Susan takes the $5,000 because we have clear and
convincing evidence that Elsie intended to modify the will by making the alteration.

________ When Elsie crossed out the "$2,000" before writing in "$5,000," she revoked
the $2,000 legacy by physical act. Is Susan nonetheless entitled to the
$2,000?

________ Suppose that Elsie, after crossing out the $2,000, writes in "$500" above it.
Does Susan get the crossed-out $2,000 under dependent relative revocation?
[What was Elsie telling us when she crossed out "$2,000" and wrote in "$500"?]

________ Suppose Elsie crossed out "$2,000" and wrote in "$5,000" immediately before
the will was signed and witnessed. Are the changes valid?

IV. WILL BENEFICIARY DIES DURING TESTATOR'S LIFETIME


A.

LAPSED GIFTS AND ANTI-LAPSE STATUTES


19. Tom executes a will that provides: "I devise Blackacre to my son Sam, and I leave my
residuary estate to my sister Mary." Sam dies two years later, survived by his wife Wendy
and his son Junior. Sam leaves a will that devises "all my property" to Wendy. Tom died two
months ago, survived by Sams wife Wendy, Sam Jr., and Toms sister Mary. Who takes
Blackacre?

a. When a will beneficiary predeceases the testator, the gift _____________________

LAW SCHOOL LEGENDS

WILLS AND TRUSTS Page 17

b. Unless the gift is saved by the state's anti-lapse statute.


These statutes vary in the scope of cases to which they apply. Some anti-lapse
statutes (e.g., Illinois) are very narrow in their operation, and apply only when
deceased beneficiary was a child or other descendant of the testator. The UPC
anti-lapse statute applies when the predeceasing beneficiary was a grandparent
or descendant of a grandparent of the testator. Several statutes are much even
broader, and apply when the predeceasing beneficiary was a relative of the
testator (e.g., Massachusetts), or a relative of the spouse or former spouse of the
testator (e.g., California).
The predeceasing beneficiary must have been within the scope of the statute,
AND must have left descendants who survived the testator.
c. Therefore, Blackacre, devised to son Sam, passes to:

d. But what of the fact that Sam left a will devising "all my property" to Wendy???
(Doesn't the anti-lapse statute save the gift for the deceased beneficiary's estate?)

e. What if Sam was not survived by descendants, meaning that the anti-lapse statute
doesn't apply; who would take Blackacre?

If a bequest or devise lapses and the anti-lapse statute does not apply, the lapsed gift:

f.

Suppose the will gave Blackacre "to my son Sam if he survives me."
________ Would the anti-lapse statute apply in favor of Junior (majority rule)?

B.

120-HOUR RULE
20. In a state that has enacted the UPCs 120-hour rule, Tim's will provides: "I give all of my
Microsoft common stock to my sister Sarah, and my residuary estate to my mother Macree."
Tim and Sarah are fatally injured an automobile accident. Tim is pronounced dead at the
scene of the accident; Sarah dies two days later. Sarah is survived by her daughter Donna
and her mother; Sarah leaves a will that devises "all my property" to the Red Cross. Who
takes the Microsoft stock under Tim's will?
________ Does the 120-hour rule apply to wills?

Page 18 WILLS AND TRUSTS

LAW SCHOOL LEGENDS

A will beneficiary who fails to survive the testator by 120 hours is treated as if he
predeceased the testator (absent contrary will provision).
Therefore, the Microsoft stock passes:

20a. Same facts as in #20, except that Tim's will provides: "I bequeath all of my Microsoft stock
to my sister Sarah if she survives me." Who takes the AT&T stock?
________ Does the 120-hour rule apply (majority rule)?

If a will contains language dealing explicitly with simultaneous deaths, deaths in a


common disaster, or requires that the devisee survive the testator in order to take,
120-hour rule DOES NOT APPLY.

C.

LAPSE IN RESIDUARY ESTATE SURVIVING RESIDUARY BENEFICIARIES RULE


21. "I bequeath my residuary estate in equal shares to my good friend Hobie Gates, my neighbor
Bill Baker, and my son Charlie. I intentionally make no provision for my son Stephen, as I
have not seen him for fifteen years." Hobie predeceases T, leaving a child (Hobie Jr.) who
survives T. T, a widower, is also survived by Baker, Charles and Stephen. Who takes the
residuary estate?
________ Does the anti-lapse statute apply in favor of Hobie Jr.?

Who takes Hobie's one-third share of the residuary estate, then?

If the residuary estate is devised to two or more persons and the gift to one of them
fails for any reason, the other residuary devisees take the entire residuary estate, in
proportion to their interests in the residue (absent contrary will provision).
21a. What if, in #21, it was T's son Charlie who predeceased T, leaving a child (Junior) who
survived T? Hobie Gates and Bill Baker survived T. Who takes Charlie's share of the
residuary estate?
The anti-lapse statute ______________________________ the "surviving residuary
beneficiaries" rule.

D.

CLASS GIFTS
22. Ted's will devises Blackacre "to the children of my good friend, Joe Barnes," and devises his
residuary estate to his wife Rose. At the time the will is executed, Joe has two children:
Andy and Bill. After the will is executed but before Ted's death, another child (Carol) is born

LAW SCHOOL LEGENDS

WILLS AND TRUSTS Page 19

to Joe, and Andy dies survived by a son, Andy Jr. Then Ted dies and his will is admitted to
probate. Eighteen months later Joe has another child, Donny. Who owns Blackacre?

Andy?
Class gift rule of construction: In a gift by will to a class of persons ("children,"
"brothers and sisters," etc.) if a member of the class predeceases the testator, the CLASS
MEMBERS WHO SURVIVE THE TESTATOR TAKE (absent a contrary will
provision). Basis: testator was "group-minded" in making the gift to the class and wanted
this group and only this group to share the property. [You read the will, and determine
the takers of the class gift, as of testator's death.]
Compare gifts to individuals: Suppose Ted's will devised Blackacre "in equal shares to
Andy, Bill and Carol, the children of my good friend Joe Barnes." Andy predeceases
Ted. The one-third share bequeathed to Andy lapses, and falls into the residuary estate as
undisposed-of property. The residuary beneficiaries would own a share of Blackacre
along with Bill and Carol.
Subject to: possible application of the anti-lapse statute. E.g., if the disposition were "to
the children of my son, Joe Barnes," since the beneficiary who predeceased the testator
was within the degree of relationship covered by the anti-lapse statute, and since he left a
child who survived the testator, Andy Jr. would take under the anti-lapse statute. The
class gift rule gives way to the anti-lapse statute when the predeceasing class member is
within the degree of relationship called for by the anti-lapse statute.
Why is Donny excluded from sharing in the gift, when he's a child of Joe Barnes???
Rule of convenience ["class closing" rule]: Rule of construction used to define the takers
of a class gift. The class is closed, meaning that later-born class members do not share in
the gift, when some class member is entitled to a distribution. This is done in order to
determine the minimum share of each class member, so a distribution can be made
without the necessity of rebate. It's called the Rule of Convenience because any other
result would be inconvenient.
Outright gift by will: the class closes at T's death.*
* Subject to gestation principle.
Common law presumption: 280 days from conception to birth

V. CHANGES IN FAMILY AFTER WILL IS EXECUTED


A.

TESTATOR MARRIES AFTER WILL IS EXECUTED


23. John executes a will that devises Blackacre (valued at $100,000) to his son Sam and his
residuary estate to his sister Sue. A year later, John marries Marsha; John dies ten months
later, leaving a $600,000 estate (which includes the value of Blackacre). John is survived by
Marsha, Sam, and his sister Sue. Who takes John's estate?
Under the omitted spouse statute found in many states, marriage following execution of will:
Omitted spouse takes _______________________________

Page 20 WILLS AND TRUSTS

LAW SCHOOL LEGENDS

UNLESS:
(i)

It appears that omission was intentional; or

(ii)

provision was made for the spouse by transfers outside the will and it is
shown that the transfers were intended in lieu of testamentary gifts by
testator's declarations, by amount of the transfer, or otherwise. [E.g., John
named Marsha as beneficiary of a $100,000 life insurance policy; or as
beneficiary of a revocable trust.]

23a. Consider the same facts, except that the above events took place in a jurisdiction that has
enacted the Revised Uniform Probate Code. Under the Revised UPC, the new spouse is
entitled to receive an amount equal to an intestate sharebut only as to that portion of the
estate not devised to testator's children from an earlier marriage; i.e., children who were
born before the marriage (or their descendants) and who are not the surviving spouses
children. Thus:
$600,000
-100,000
$500,000

Johns estate
value of Blackacre bequeathed to son Sam
amount subject to Marshas share as omitted spouse

Reviewing the Revised UPCs intestate distribution rules (see p. 2), Marsha would be
entitled to:
$100,000
+200,000
$300,000

first $100,000John was survived by descendants not Marshas descendants


plus of balance
Marshas share as omitted spouse

The remaining estate (property worth $200,000) would pass under the wills residuary
clause to sister Sue.
B.

TESTATOR IS DIVORCED AFTER WILL IS EXECUTED


24. Hank is married to Wendy, who has a child (Wookie) by her first marriage. Hank writes a
will that devises Blueacre to Wendy, and the rest of his estate to his sister Sue. The will
names Wendy as executor "if she is able; otherwise my brother Sam is to serve as executor."
Two years later Hank and Wendy divorce; then Hank dies without having changed his will.
Who takes Blueacre? ____________________________________
Who serves as executor? _________________________________
A final decree of divorce or annulment revokes all gifts and fiduciary appointments
in favor of former spouse. The will is read and the estate is distributed (and
fiduciaries are named) as though former spouse predeceased the testator.
________ But doesn't the anti-lapse statute apply in favor of Wendys child Wookie,
then?

LAW SCHOOL LEGENDS

WILLS AND TRUSTS Page 21

But if the couple divorces and then remarries, so that W is H's wife at death, she takes
under the will. The statute operates to revoke gifts and appointments only if they are
divorced at testator's death.

C.

TESTATOR HAS CHILD AFTER WILL IS EXECUTED -- "PRETERMITTED CHILD"


25. When he made his will, Tank and his wife had one child: Alvin. After Tank made the will he
and his wife adopted a child: Billy. Tank died survived by his wife, Alvin and Billy. His
will left two-thirds of his $260,000 estate to his wife and the other one-third to a sister. What
rights if any do the children have in this estate and why?
Alvin (alive when the will was executed):

Unless [says UPC] omission of the child was not intentional but was by accident or
mistake. [E.g., T mistakenly thought Alvin was killed in Viet Nam. Statute applies
only to mistake as to child of testator. If he mistakenly thought his brother was
killed in Viet Nam, UPC statute does not apply.]
Billy? Pretermitted child (born or adopted after the will was executed) takes:

Unless it appears from the will (no extrinsic evidence) that omission was
intentional.
25a. Same facts, except that two years after adopting Billy, Tank executes a codicil to his will
that names Second Bank rather than First Bank as executor.

Under the doctrine of republication by codicil, the will speaks (is republishedis
deemed to have been executed) on the date of the last codicil thereto. Billy is treated as
having been born before the will was executed, and has no rights as a pretermitted child.

VI. PROBLEMS ASSOCIATED WITH TYPES OF TESTAMENTARY GIFTS


Specific devise or bequest: "I devise Blackacre to my son John."
Demonstrative legacy:

A general amount from a specific source. "I bequeath $25,000,


to be paid from the proceeds of the sale of my Shell Oil stock, to
Sally."

General legacy:

"I bequeath the sum of $10,000 to my nephew Ned."

Residuary gift:

"I give all the rest, residue and remainder of my estate to Betty."

Intestate property:

If a partial intestacy because the will, poorly drafted, does not


contain a residuary clause.

Page 22 WILLS AND TRUSTS

A.

LAW SCHOOL LEGENDS

ABATEMENT OF LEGACIES TO PAY DEBTS AND EXPENSES


What happens when there are so many claims against the estate that there are not enough assets to
cover all of the gifts made by the will? ("Abatement" problem.) Absent contrary provision, debts
and expenses are first paid out of:
(1) intestate property, if a partial intestacy for some reason;
(2) then out of residuary assets,
(3) then out of general legacies, and finally
(4) out of specific bequests.
Within each category, gifts abate pro rata; no distinction is made between real and personal
property. (Nearly all states have abolished the common law rule under which personal property
was sold off before any real property was touched.)
Under the UPC and in most states, demonstrative legacies are treated the same as specific bequests
(and thus last to be abated) to the extent of the value of the specified property, and as a general
legacy to the extent of any excess. If (e.g.,) the will made a bequest of $25,000, to be paid from
the proceeds of the sale of my Shell Oil stock, to Sally," but the Shell stock was only worth $16,000
at testators death, for abatement purposes it would be treated as a specific bequest as to $16,000,
and as a general legacy as to $9,000.

B.

PRO RATA APPORTIONMENT RULE GOVERNS DEATH TAXES


26. Counting life insurance, Ted has a $2,000,000 estate (net of expenses and debts). Ted's will
bequeaths Blackacre (worth $400,000) to his son Al, $200,000 to his church, and his
residuary estate ($1,200,000 after debts and expenses) in trust: Income to Ted's daughter
Betty for life, remainder to Betty's children. Ted was the insured under a $200,000 life
insurance policy; the proceeds are paid in a lump sum to Ted's sister Carol. Allowing for a
$200,000 charitable deduction, Ted's taxable estate was $1,800,000, and estate taxes payable
by reason of Ted's death total $108,000. Ted's executor wants to know: Against whom
should the $108,000 in death taxes be charged? Advise him.

Majority rule: Absent contrary will provision, death taxes are apportioned (on a pro rata
basis) among all persons interested in the estate (beneficiaries of both probate and nonprobate transfers).
Exception: Beneficiary of an interest that qualifies for an estate tax charitable or marital
deduction gets benefit of that deduction (does not have to contribute pro rata).
Numerator:
Denominator:

value of each testamentary & nontestamentary gift


total value of taxable estate

Al [specific devise of Blackacre]:


Church [$200,000 cash legacy]:

$400,000 x $108,000 = $24,000


$1,800,000

LAW SCHOOL LEGENDS

C.

WILLS AND TRUSTS Page 23

Residuary trust:

$1,200,000
$1,800,000

Carol [$200,000 life ins. proceeds]:

$200,000_ x $108,000 = $12,000


$1,800,000

x $108,000 = $72,000

SPECIFICALLY DEVISED PROPERTY NOT IN ESTATE AT DEATH -- ADEMPTION


27. Terry executes a will that devises Blackacre to his brother Bob, and his residuary estate to his
wife. Two years later, Terry sells Blackacre for cash, and uses the sale proceeds to buy
Whiteacre. Terry died last month without having changed his will. What does Bob take
under Terry's will?
________ Where the will makes a specific gift of property, and the property is not
owned by the testator at death (for whatever reason):

28. Henny's will contains this provision: "I bequeath the sum of $25,000, to be paid out of the
proceeds of sale of my Shell Oil stock, to Sally." [Demonstrative legacy.] One year before
her death, Henny sells all of her Shell Oil stock and uses the sale proceeds to buy Exxon
stock.
________ Does ademption apply to demonstrative (or general) legacies?

On Henny's death, what does Sally get?

(But if Henny owned any Shell Oil stock at her death, executor would be under duty
to sell it to raise the $5,000.)

D.

STATUTORY EXCEPTIONS TO THE ADEMPTION DOCTRINE


29. Same facts as in #27 (specific devise of Blackacre), except that Terry sold Blackacre for
$10,000 cash and a $90,000 note secured by a mortgage on Blackacre. At Terrys death, the
balance on the note is $65,000. What does Bob take under Terry's will?

These UPC provisions, which have been enacted in a number of non-UPC states, reverse the
common law rules, which applied ademption to any case where specifically devised property was
not in the estate at death. (At common law, testators probable intent was immaterial.)
#1.

Specific devisee takes any remaining specifically devised property and:

Page 24 WILLS AND TRUSTS

LAW SCHOOL LEGENDS

--

Any unpaid balance of purchase price (together with any security interest) by
reason of sale of the property.

--

Any amount of condemnation award for taking of the property, to the extent
unpaid at testator's death.

--

Any amount of fire or casualty insurance proceeds for damaged or destroyed


property unpaid at death.
Caveat: These rules DO NOT APPLY if the sale proceeds, condemnation
award or insurance claim were fully paid before testator's death. (Rationale:
Testator had time to change his will.)

--

#2.

E.

Property acquired as a replacement property for specifically devised property.


[I devise my residence on Smith Street to my sister Sue; T sells that house and
buys a residence on Oak Street.]

Will executed before T declared incapacitated: If specifically devised property is sold


by guardian or conservator, or if condemnation award or insurance proceeds are paid
to the guardian or conservator because of fire or casualty, the specific devisee has a
right to a general legacy equal to the net sale price, condemnation award, or
insurance proceeds. (Otherwise, guardian or conservator could change the will by
deciding what assets to sell. Also, T didn't have capacity to change his will to adjust
for the loss or destruction of the property.)

BEQUESTS OF STOCK AND OTHER SECURITIES


30. Tony executes a will that contains the following clauses:
"5th. I give my 100 shares of IBM stock to Albert Avins."
"6th. I give 100 shares of Kodak stock to Ben Baker."
One year before his death, Tony sells his IBM stock and uses the sale proceeds to buy AT&T
stock. Also before his death, Tony sells his Kodak stock and invests the proceeds in 200
shares of Polaroid stock.

a. What does Avins get ("my 100 shares of IBM stock")?

b. What does Baker get ("100 shares of Kodak stock")?

For ademption purposes,

LAW SCHOOL LEGENDS

WILLS AND TRUSTS Page 25

31. Suppose, instead, that Tony does not sell his Kodak stock? After Tony executes the will but
before his death, Kodak splits two-for-one, and certificates for 200 shares of Kodak stock are
found in Tony's safe deposit box after his death.
[Majority rule: A specific bequest of stock includes any stock produced by a stock split,
stock dividend, and stock resulting from a merger, reorganization, or other action
initiated by the entity after the will was executed, but does not include stock acquired by
the exercise of a stock option.]
Held: Baker takes all 200 shares of Kodak stock, because the gift of stock was a specific
bequest!
But how can that be, when the gift was called a general legacy in #30?? Something is going on
herebut let us recognize where it is. In #30, the courts have seized on the absence of a possessive
pronoun "my," calling it a general bequest to avoid ademption. But the issue here is different (stock
split). A gift of stock can be specific for one purpose (stock split) and general for another purpose
(ademption).
F.

SPECIFIC GIFT OF ENCUMBERED PROPERTY IS THE LIEN EXONERATED?


32. Teddy's will devises Whiteacre to Joan, and devises his residuary estate to Betty. On Teddy's
death, Whiteacre is subject to a mortgage securing a $10,000 note on which Teddy was
personally liable. Joan demands that Teddy's executor pay the $10,000 debt out of the
residuary estate so that she will take title free of the mortgage lien. Is she entitled?
Common law:

YES because liens on specifically devised property are "exonerated


from the residuary estate.

UPC and majority rule: ________* because exoneration of liens doctrine:


(*unless the will directs exoneration)

Under the majority rule, Joan takes:

VII. REFERENCE TO FACTS AND EVENTS OUTSIDE THE WILL


A.

INCORPORATION BY REFERENCE DOCTRINE


33. On July 1, 1998, Thor executes a will that provides: "I devise Blackacre to the person named
in a memo dated May 4, 1998, that I have written and placed in my safe deposit box. I give
my residuary estate to my brother Ben." After Thor's death there is found, in his safe deposit
box, a typed memo dated May 4, 1998, signed by Thor but not witnessed: "Pursuant to my
will, I want Blackacre to go to my nephew Norman."
________ Does Norman take Blackacre under the memo, when the memo was not
witnessed, and was not part of the will that was signed by Thor and the
attesting witnesses?

Page 26 WILLS AND TRUSTS

LAW SCHOOL LEGENDS

An extrinsic document, not present when the will was executed (and thus not part of the duly
executed will), can be INCORPORATED BY REFERENCE into the will . . . IF:
#1.

Writing must be in existence when the will was executed.

#2.

Will must show an intent to incorporate the writing.

#3.

Will must describe the writing sufficiently to permit its identification. ("So there can be
no mistake as to the identity of the document referred to.")

33a. Same facts, except that the memo was dated May 4, 1999.

________ Incorporation by reference on these facts?

34. With Tom's will in his safe deposit box was the following typewritten, unwitnessed memo
written after Tom signed his will: "In my will I referred to a list that I would prepare at a
later date leaving certain items of personal property, and this is it: I leave my golf clubs to
my friend, Hobie Gates, my fishing tackle to my son Sam, $2,000 to my daughter Donna, and
my IBM stock to my brother Ivan. /s/ Tom Testator." If the state has enacted the controlling
UPC provision, valid disposition . . .

________ as to golf clubs?

________ as to the fishing tackle?

________ as to $2,000?

________ as to the IBM stock?

Under the UPC and several non-UPC states, statutory exception to incorporation by reference rule:
Will may refer to written statement or list that disposes of TANGIBLE PERSONAL PROPERTY
(other than money, intangibles, property used in trade or business) not specifically disposed of by
the will. The written list must be signed by testator, must describe the property with reasonable
certainty. Can be written before or after the will is executed; can be altered at any time.
Statute provides a simple and inexpensive procedure for making gifts of personal items of
sentimental value, without having to amend the will every time the client changes his mind, or
wants to add to the list.
B.

ACTS OF INDEPENDENT SIGNIFICANCE DOCTRINE


35. Tina dies leaving a will that provides: "I give the automobile that I own at my death to my
nephew Norman. I give the furniture and furnishings in my living room to my sister Sue." A
year before her death, Tina had traded her 1999 Honda Civic in on a brand new Mercedes
Benz. Six months before her death, Tina moved a $25,000 Picasso print from his den and
mounted it on her living room wall. What is the effect of these acts on the provisions of
Tina's will?
________ Does Norman take the Mercedes? ________ Does Sue take the Picasso?

LAW SCHOOL LEGENDS

WILLS AND TRUSTS Page 27

Key words: Acts of independent significance


Also known as doctrine of nontestamentary acts
*________ Same result for gift of "contents of my sea chest"?
* Except for title documents: deeds, stock certificates, bank passbooks

VIII. OTHER WILLS DOCTRINES


A.

MISTAKES OR AMBIGUITIES IN THE WILL


36. Tom tells his lawyer to draw his will and give his nephew Ed "300 shares of Acme stock." In
typing the will, the typist makes a mistake and types the figure as "200," which Tom does not
notice when he signs the will. At Tom's death he owns 300 shares of Acme stock. Ed offers
the testimony of Tom's lawyer that the typist made a mistake. What does Ed get and why?
________ shares:
Absent suspicious circumstances, it is conclusively presumed that:
37. "I give $10,000 to my nephew, John Paul Jones." Problem: T has a nephew James Peter
Jones, and another named Harold Paul Jones, but no nephew named John Paul Jones. Who
takes the $10,000?
This is called a _____________________________________ because there is
a ________________________________________.

________ Is extrinsic evidence admissible?

What if the extrinsic evidence does not cure the ambiguity?

38. Rod's will includes this gift: "I give the sum of Twenty-five Dollars ($25,000) to my niece
Nora."
This is called a ______________________________________________________.

________ Is extrinsic evidence admissible?


________ Does the admissible evidence include "facts and circumstances"

Page 28 WILLS AND TRUSTS

LAW SCHOOL LEGENDS

evidence? (evidence about the testator, his family, the claimants under the
will and their relationship to the testator, testator's habits and thoughts,
etc.)
________ [majority rule:] Does the admissible evidence include testator's
declarations of intent to a third party? (E.g., he told a friend he had
bequeathed $25,000 to his niece Nora.)
[But modern trend is to admit all types of extrinsic evidence to cure patent as well as latent
ambiguities.]
________ Does the admissible evidence include anything testator said to his
attorney?

B.

CONTRACTS RELATING TO WILLS


Under the UPC and in most states by statute, a contract to make a will or not to revoke a will, can
be established only by:
(1) provisions in the will stating the material provisions of the contract, or
(2) express reference in the will to a contract and extrinsic evidence proving the contract's
terms, or
(3) a writing signed by decedent evidencing the contract. Execution of a joint will or
reciprocal wills does not raise a presumption that a contract exists.
These statutes have eliminated troublesome litigation over joint wills (the wills of two persons on
one piece of paper -- "We and each of us dispose of our property as follows. . ."), as to whether the
will (wills?) was executed pursuant to a contract that the surviving party will not revoke the joint
disposition. The cases sometimes found the existence of a contract merely from the execution of a
joint will using plural possessive pronouns (we, us, our) that made a disposition of the combined
estates.

C.

EFFECT OF WORDS OF DISINHERITANCE IN A WILL


39. Tammy's will devises Redacre to her son Sam and her residuary estate to her husband Harold.
The will provides: "I intentionally make no provision for my daughter Nancy, as she married
out of the faith and has been a great disappointment to me." Two years later, Tammy
divorces Harold, and two years after that Tammy dies without having changed her will. She
is survived by Sam and Nancy as her nearest kin. Nancy had no children. Who takes the
residuary estate?
Harold?

Who takes the residuary estate, then?


___________ to Nancy.

(majority rule) ___________ to Sam and

LAW SCHOOL LEGENDS

WILLS AND TRUSTS Page 29

Most states apply the common law rule, which says: When a will does not make a complete
disposition of the estate (partial intestacy), words of disinheritance in the will are ineffective.
Rationale: When property passes by intestate succession, it passes pursuant to the intestacy
statute, not the decedent's will.
"However, under the Uniform Probate Code's negative bequest rule, a will can provide
how property shall NOT be disposed of, meaning that words of disinheritance are given
effect; estate is distributed as though disinherited person predeceased the testator.
Under the UPC, Sam would inherit the entire residuary estate.

D.

NONPROBATE ASSETS are interests in property that are not subject to disposition by will or
inheritance, and do not pass through a person's probate estate for purposes of administration. Major
types (also called nontestamentary assets):
#1.

Property passing by right of survivorship (joint bank account, etc.).

#2.

Property passing by contract: life insurance, employee retirement benefits paid to


beneficiary other than insured's executor or estate.

#3.

Property held in trust, including a revocable trust, where trust terms govern
distribution of assets.

#4.

Property over which the decedent held a power of appointment.

40. T has a $50,000 Aetna life insurance policy that names Bill Bates as beneficiary. T dies
leaving a will that provides: "I direct that the proceeds of my Aetna life insurance policy be
paid to my sister Ann Painter." Who takes the $50,000 policy proceeds?

IX. ELECTIVE SHARE STATUTE


41. Winkie dies leaving a will that provides: "I give my husband Hobie Gates the sum of $23,
which is one dollar for each miserable year I spent with him. I give all the rest of my
property to my faithful chauffeur Claude, in appreciation of his many services." Winkie is
survived by Hobie, Claude and the couples daughter Dorkie. What are Hobie's rights?
Hobie should file for an elective share of Winkies estate. All non-community property
states except Georgia have elective share statutes designed to protect spouses against
disinheritance, by ensuring that the surviving spouse can take a specified minimum share of
the decedents estate. (None of the community property states has an elective share statute.)
1. Amount of elective share. In many states, the elective share amount is one-third of
the decedent's net estate if the decedent was survived by descendants, one-half if the
decedent was not survived by descendants.
Under the original (1969) Uniform Probate Code, the amount of the elective share is
one-third of the net estate, regardless of whether the decedent was survived by
descendants.

Page 30 WILLS AND TRUSTS

LAW SCHOOL LEGENDS

The Revised 1990 UPC takes an accrual approach (c.f. pension plan vesting rules),
under which the amount of the elective share is tied to the length of marriage. The
surviving spouse is entitled to an elective share percentage of 3% per year for the
first 10 years of marriage, and 4% for the next 5 years until after 15 years of
marriage the elective share is fully phased in at of the estate.
2. Spouse must file notice of election. The right to an elective share is not automatic.
The spouse must file a notice of election within a specified period (usually within 6
months after the will is admitted to probate).
3. Who may make the election. Election can be made on behalf of a legally
incapacitated spouse by a guardian or conservator, with court approval, upon a
showing that an election is necessary to provide adequate support for the spouse
during his probable life expectancy. But if the spouse dies before election is made,
an election cannot be made by the deceased spouse's personal representative.
[Rationale: Purpose of elective share is to protect the surviving spouse against
disinheritance, not to provide benefits to the spouse's heirs.]
4. In making up the elective share all beneficiaries of the estate contribute pro rata, and
their interests in the estate are reduced pro rata. However, property left outright to
the spouse by will is first applied. (Purpose: To avoid disruption of decedent's
testamentary plan as far as possible.) If, for example, Rick's will devises Blackacre
to his wife Lucy and the rest of his estate to his daughter, and Lucy files for an
elective share, the value of Blackacre is first applied in making up the elective share
entitlement.
42. Two years before his death, Hank established a revocable trust naming Acme Bank as trustee:
Income to Hank for life, and on Hank's death remainder to his son Steve (by a former
marriage). Hank died two months ago, leaving a will that bequeathed his estate to his wife
Wendy and Steve in equal shares. While that sounds fair enough, nearly all of Hank's property
had been placed in the trust. Hank left a net probate estate (after expenses) of $36,000; the
value of the assets in the revocable trust at Hank's death is $900,000. Wendy files for an
elective share. Does Wendy's elective share right apply to the assets in the revocable trust?
Minority rule: NO. Elective share statute giving one-third or one-half of the decedent's
"estate" means the probate estate. The elective share does not apply to non-probate
transfers such as revocable trusts, etc., because these are not part of the transferor's
"estate."
UPC and majority rule: YES. The policy underlying the elective share should not be
defeated by lifetime and nonprobate transfers in which the donor retains rights, powers, or
economic benefits. In most states, the elective share applies to the AUGMENTED
ESTATE, which includes the net probate estate and also lifetime transfers in which the
grantor retained the power to revoke, or to invade, consume or dispose of principal. In
addition to revocable trusts, this includes Totten Trust ("A, Trustee for B") bank accounts,
joint and survivor bank accounts, etc.

X. WILL CONTESTS
Only interested parties can bring a will contest: Persons with an economic interest adversely
affected by the will's probate. (Heirs, legatees under earlier will whose interest would be defeated

LAW SCHOOL LEGENDS

WILLS AND TRUSTS Page 31

by this will.) Thus a close personal friend, not named as legatee in an earlier will, has no standing
to contest the decedent's will.
A.

LACK OF TESTAMENTARY CAPACITY.


The test: Did the testator have sufficient capacity to:
1.

Understand the nature of the act he was doing? (He was writing a will.)

2.

Know the nature and approximate value of his property?

3.

Know the natural objects of his bounty?

4.

Understand the disposition he was making?

Evidence of capacity or lack of it must relate to circumstances at the time the will was executed, or
shortly before or shortly thereafter. The more distant in time from the will's execution a particular
fact may be, the less significance it has on the question in issue: Did the testator, at the time the
will was executed, have capacity? [Recent case: Six months before signing his will, T had been in
a mental hospital suffering from paranoia and manic depression: Evidence was too remote to be
relevant to condition at the time the will was signed.]
Mere old age, poor health, frailty, failing memory, or vacillating judgment are not inconsistent with
testamentary capacity if the testamentary prerequisites [above] were possessed by testator.
43. T was 93 years old when he signed his will. Six months earlier, T had been adjudicated
incapacitated, and a guardian [or conservator] was appointed to manage his property. The
trial judge granted the heirs' motion for a directed verdict on the ground that T did not have
testamentary capacity.
________ Was this decision proper?
#1: Adjudication of incapacity involves ______________________________
(capacity to contract, to manage one's affairs).
#2. Jury could find that the will was executed during a ______________________.
B.

UNDUE INFLUENCE: Where one with testamentary capacity is subjected to and controlled by a
dominant influence or power. Burden of proof is on contestants, who must show:
1.

Existence and exertion of the influence.

2.

Effect is to overpower the mind and will of the testator.

3.

Product is a will (or a gift therein) that would not have been made but for the
influence. (Undue influence may be shown as to the entire will, or as to one gift in the
will.)

"Influence is not undue unless the free agency of the testator was destroyed and a will produced
that expresses the will, not of the testator, but of the one exerting the influence." ["mental duress"]
While evidence of undue influence is usually circumstantial, these alone are not enough:
1.

Mere opportunity to exert influence. Fact that one child (who received major share of

Page 32 WILLS AND TRUSTS

LAW SCHOOL LEGENDS

the estate) lived with mother, wrote checks for her, balanced the checkbook, helped on
income tax, held a power of attorney . . . is not evidence that the opportunity was taken
advantage of.
2.

Mere susceptibility to influence due to illness, age. Fact that Mother was very old, had
broken her hip, had memory lapses, took Valium . . . this is not evidence of undue
influence.

3.

Mere fact of unnatural disposition Fact that will gave daughter of estate and her
two sons one-sixth eachthat is not evidence that the will was the product of undue
influence.

44. Tommy's girlfriend Gloria nagged him, badgered him, and threatened to leave him unless he
"proved his love" by writing a will in her favor, which he did. Undue influence?

45. Will drafted by Attorney's law partner devised entire estate to Attorney, whom client had never
met before.
Where there exists a confidential relationship between the testator and a party, and that party
will benefit from the will, and that party had the opportunity to exert undue influence,
there is an _________________________________________________ of undue influence,
which is strengthened when there are ___________________________________________
If an inference is raised, this doesn't affect the burden of proof (contestant still has burden of
proof), but will proponent now has the burden of going forward with evidence that no undue
influence was exerted. If will proponent does not produce sufficient rebuttal evidence, the
inference satisfies the contestant's burden of proof on the issue of undue influence.
46. Terry (who has no children) leaves a will that bequeaths $10,000 to his niece Nell and his
residuary estate to his cousin Sam. The will contains a no contest ("in terrorem") clause: "If
any beneficiary contests this will or any part thereof, he shall forfeit any interest given to him
by my will." Nell contests the will on grounds of incapacity and undue influence, but loses;
the will is admitted to probate. Does Nell forfeit her legacy?
___________ . . . unless she had ______________________________________________.
Majority rule: No-contest clauses are given full effect unless the court finds that the contest
was brought in good faith and with probable cause (i.e., it wasn't a strike suit designed to
extract a settlement). Thus Nell does not forfeit the legacy IF court finds she had probable
cause for filing the will contest.
THE "UNLESS" IS THE MOST IMPORTANT PART OF THE MAJORITY RULE!
In several states (e.g., Massachusetts, New York), no-contest clauses are given full effect
regardless of whether the contest was filed with probable cause. Rationale: A testator

LAW SCHOOL LEGENDS

WILLS AND TRUSTS Page 33

should be permitted to protect his testamentary plan, and his reputation, against post-death
attack.

XI. ESTATE ADMINISTRATION -- CREDITORS' CLAIMS


47. Tom died May. His will was admitted to probate, and his wife Wilma, named as executor in
the will, published proper notice to creditors (in a newspaper of general circulation) on June
10. For the last two months of his life, Tom was hospitalized at Mercy Hospital. On
December 20, the Hospital filed its claim against Tom's estate for the $24,000 expenses of
Tom's last sickness. Wilma as executor refuses to pay the claim.
________ [Former law:] Is Hospital entitled to collect the $24,000 from Tom's estate?

________ [Current law:] Is Hospital entitled to collect the $24,000 from Tom's estate?
Traditionally, nearly all states had nonclaim statutes (special short statutes of limitation) applicable
to claims against a decedents estate. All claims not presented within a prescribed period of time
after the first publication of notice of administration [e.g., Illinois, 6 months, Uniform Probate
Code, 4 months; Oklahoma, 2 months) were barred.
In Tulsa Professional Collection Service v. Pope (1988), the U.S. Supreme Court held that the
Oklahoma nonclaim statute (which provided for notice by publication only) was unconstitutional as
applied to known creditors or creditors who were reasonably ascertainable. The Due Process
clause requires personal notice to known creditors before their claims can be barred. As a result,
all states with similar nonclaim statutes had to amend their statutes.
The Uniform Probate Code has three provisions relating to creditors claims. Under UPC 3-801(b),
the personal representative may (its optional) give personal notice to creditors, requiring them to
present their claims within the later of (i) 60 days after receipt of the notice or (ii) 4 months after
the first publication of notice; otherwise their claims will be barred. The personal representative
who wants to expedite matters can give personal notice to any creditor, and can shorten the time
required for presenting the claim.
Under UPC 3-801(a), all claims not presented within 4 months after the first publication of notice
of administration are barred. This the very type of statute that was ruled unconstitutional in Tulsa
Professional Collection Service v. Popebut only as applied to known or ascertainable creditors.
This UPC provision recognizes that it is still constitutionally permissible to cut off the claims of
unknown and unascertainable creditors with notice by publication only.
UPC 3-803 sets out a one-year statute of limitations applicable in all cases, whether the decedent
left a will or died intestate, and even if no permissive personal notice or notice by publication was
given. All claims not presented within one year of the decedents death are barred. Dictum in
Tulsa Professional Collection indicated that such a self-executing statute of limitations, which
applies to all cases and is not dependent on any probate court, would be constitutionally
permissible.

Page 34 WILLS AND TRUSTS

LAW SCHOOL LEGENDS

TRUSTS
TRUST is an arrangement for making gifts of property and for the management of assets, under which
the trustee holds legal title to the trust assets for the benefit of the beneficiaries, who hold equitable title.
Trustee has the burdens of ownership (duty to manage, safeguard, invest, etc.); beneficiaries have
equitable title and the benefits of ownership.

I. REQUIREMENTS FOR A VALID TRUST


To have a valid trust, SETTLOR . . . DELIVERS . . . RES (trust property) . . . to TRUSTEE . . . for the
benefit of the BENEFICIARIES, with INTENT TO CREATE A TRUST. Trust must be for a
LAWFUL PURPOSE. As with the law of gifts, no consideration is required for creation of a trust.
A.

SETTLOR must have legal capacity. (must be age 18 or over; must have capacity to convey legal
title to the trustee, a higher test for capacity than for wills.)

B.

DELIVERY requirement does not apply to a self-declaration of trust ["I hereby declare myself
trustee..."] or testamentary trust. But for an inter vivos trust that names third party as trustee, the
mere intent to create a trust, or a gratuitous promise to create a trust, is not sufficient. As with the
law of gifts there must be delivery of subject matter of the trust, with the intent to convey legal title
to the trustee.
1. Sam told lawyer he wanted to create trust for his daughter, but died before signing trust
instrument or delivering assets to the trustee. Valid trust?
________ because no delivery of the trust assets with intent to transfer title during
Sam's lifetime.

Same answer where trust signed by Dad purported to create trust to be funded with
"whatever money or property that I contribute to the trust over the next ten years." This
was merely a promise to create a trust in the future, not supported by consideration.
C.

RES: the corpus, the principal, the subject matter of the trust.
To have a trust, legal title to a specific interest in property must be conveyed to the trustee. The
subject matter of the trust must be certain and identifiable. If there is no trust property, there is no
trust.
2. When Grandpa told Uncle Hobie Gates that he was leaving Whiteacre to Hobie in his will,
Hobie signed an irrevocable Declaration of Trust: "I declare myself trustee of my interest in
Whiteacre; and when I receive it from Grandpa's estate, I shall pay the income therefrom to
Nephew Ned for life, remainder to his issue." After Grandpa's death Uncle Hobie received
Whiteacre pursuant to the will. Can Nephew Ned enforce the trust against Uncle Hobie?
________ Was valid trust created when Hobie wrote the irrevocable Declaration of
Trust?
Because at that time, Hobie had an:

LAW SCHOOL LEGENDS

WILLS AND TRUSTS Page 35

A different result is reached when the promise to hold property (to be received in the
future) in trust is supported by consideration. Under contract principles, the trust
automatically attaches when the property is received.
3. Willa, who owns common stock that pays her over $100,000/year, writes, signs and notarizes
a document: "I hereby create a trust for the benefit of my maid Mimi. I shall pay Mimi
$3,000/month from dividends paid on stock that I own. After my death, [Bank] as trustee
shall continue to pay Mimi $3,000/month until her death." No cash, stock or other property
was set aside as the corpus of the trust.
________ Valid trust?
D.

TRUSTEE
Must have legal capacity to deal with the property (must be over age 18; must have capacity to
contract and to execute a deed).
4. Ann's will devises her residuary estate in trust: "income to my daughter Martha for life, and
on her death to distribute the trust principal to her descendants." However, the will does not
name anyone as trustee.
________ Valid trust?
No trust fails for lack of a trustee. If the intent to create a trust is clearly manifested but no
trustee is named or if the named trustee dies or resigns with no provision for a successor
trustee, the court will appoint a suitable successor to carry out the trust.
5. Irreconcilable conflicts arise between First Bank (the trustee) and the beneficiaries. May
First Bank resign as trustee? If so, what procedure must it follow?
#1:

#2:

5a. Same facts in #5, except that First Bank does not want to resign as trustee. The beneficiaries
bring an action seeking to remove First Bank as trustee, citing the irreconcilable conflicts.
Should they prevail?

________ . . . unless the beneficiaries can show that conflicts prevent trustee from
carrying out duties, or that trustee has breached a material term of the trust.
What explains the different results in #5 and #5a?

Page 36 WILLS AND TRUSTS

LAW SCHOOL LEGENDS

6. Albert transfers title to Blackacre to Hobie Gates as trustee for the benefit of Albert's's son
Carl. Under the agreement, Carl has the power to manage and control the use of Blackacre,
and Hobie as trustee has no powers or active duties over the property. [Cf. Statute of Uses,
1536!]
________ Valid trust with respect to Blackacre?
If named trustee has no powers or active duties to perform,

What is the state of title to Blackacre, then?

6a. Suppose the instrument in #6 contains a spendthrift clause with respect to Carl's interest: "No
beneficiary shall have the power to transfer his interest, nor shall such interest be reachable
by the beneficiary's creditors."
The spendthrift clause in #6 is _______________________________
spendthrift trust,

E.

To have a

BENEFICIARIES
For a private (noncharitable) trust, the beneficiaries must be definite and ascertainable, and their
interests must vest, if at all, within period of Rule Against Perpetuities. (Rule for charitable trusts
exactly the opposite; see below.)
7. "I bequeath $100,000 to my friend Hobie Gates as trustee, and I direct him to pay the income
therefrom to my best friends. I bequeath my residuary estate to my brother, Sam Slade."
________ Valid trust?
________ Does Hobie get to keep the money, then?
Hobie holds on a resulting trust for Sam Slade, the residuary beneficiary. [A resulting trust
is not a trust. It's the term the courts use to describe the situation where a trust fails for
some reason. The court will order that the $100,000 be distributed directly to Sam Slade.]
a. What if the instrument provided that Hobie as trustee was "to pay the income to
my family," or "to my wife's next of kin." Valid trust?

LAW SCHOOL LEGENDS

WILLS AND TRUSTS Page 37

8. $200,000 left to bank as trustee, "to use the income to train spiritualistic mediums." Valid
trust?

Cf. Estate of Kidd (Ariz. 1971): Holographic will left $240,000 estate left "for reserach [sic] or
some scientific proof of a sole of the human body which leaves at death. I think in time their
[there?] can be a Photograph of sole leaving the human body at death."
a. Testamentary capacity????
b. Valid gift?

F.

INTENT TO CREATE A TRUST: IS THE LANGUAGE PRECATORY (a non-binding


suggestion), OR DOES IT IMPOSE AN ENFORCEABLE OBLIGATION?
9. Tammy's will devises here residuary estate to her husband Harold, "but I would like him to
take care of my dear friend Fred Frypan [a longtime employee] in appreciation for Fred's
years of loyal and dedicated service." When Harold refuses to make any distributions to
Fred, Fred brings suit against Harold to compel payment for Fred's care. Result?

Other precatory words:

10. Granny gives Sonny a check for $10,000 with the notation (on the check) "for the use and
benefit of [grandson] Gary." Sonny deposits the money in his personal account and refuses
to disburse funds for Gary's benefit.
________ Was a valid trust created, given that the words "trust" and "trustee" were not
used?

G.

LAWFUL PURPOSE
Trust whose purpose is to further the commission of a crime, or calls for the destruction of
property, is invalid as against public policy. Also, unlawful conditions are unenforceable.
11. Trust "to pay the income to my daughter Betty until such time as Betty divorces her husband
Hobie Gates, at which time the trust shall terminate and all trust principal shall be distributed
to Betty. If Betty does not divorce Hobie, on Betty's death the principal shall go to the
National Organization for Women."

Page 38 WILLS AND TRUSTS

LAW SCHOOL LEGENDS

________ Valid condition?

What happens to the trust property, then?

12. Dad's will bequeaths $200,000 to his son David, "provided he marries a Jewish woman within
seven years after my death. If David does not meet this condition, the $200,000 shall go to
the State of Israel."
________ Valid condition?

Compare testamentary trust "to pay the income to my wife for life or until she remarries. If
my wife remarries, the trustee shall pay the income to my daughter Bonnie for life." Valid
condition?

Compare reputed will of Heinrich Heine, German poet: Money bequeathed to wife "on the
express condition that she remarry. I want at least one person to be truly bereaved by my
death."

II. REVOCABLE TRUSTS


13. On October 11, 2003, Winkie transfers securities worth $500,000 to Acme Bank as trustee of
a revocable, amendable inter vivos trust: Income to Winkie for life, and on Winkie's death
the trustee shall distribute the trust principal to her son Steve. The trust instrument is signed
by Winkie and an officer of Acme Bank, but is not witnessed. On the same day, Winkie
executes a will that, after making various bequests, bequeaths her residuary estate "to Acme
Bank, as trustee, to be added to and administered under the terms of the trust that I executed
on October 11, 2003."
This is a valid trust, and can be useful arrangement to provide for management in event
of settlor's future incapacity, avoiding expenses and restrictions of a guardianship
administration. As long as there are one or more trust beneficiaries besides the settlor, a
trust is not void as an attempted testamentary disposition (i.e., doesn't have to be
executed with formalities of a will) even though settlor retains any one or more of the
following rights and powers:

Income for life.

Power to revoke, alter, amend or terminate the trust.

Power to control trustee in the administration of the trust.

LAW SCHOOL LEGENDS

WILLS AND TRUSTS Page 39

Power to cause life insurance proceeds or employee benefits to be paid to the


trust.

Settlor can name herself as trustee, to serve as long as she has capacity to serve.
The testamentary gift to the trust, called a pourover will gift, provides a mechanism
for adding testamentary assets to a trust created by the testator during lifetime. By
statute, such a pourover gift is valid:
(i) even if trust is subject to revocation and amendment and is later amended,
and
(ii) even if trust is unfunded life insurance trust (eliminates concern about "res"
in an unfunded life insurance trust).
Majority rule: To be a valid recipient of a pourover gift from a will, trust must be in
existence before or executed concurrently with will.
UNIFORM TRANSFERS TO MINORS ACT provides a convenient method for making gifts to
minor children that:
(i)

avoid the need for appointing a guardian to manage the property, and

(ii)

qualify for the $11,000 per-donee annual exclusion under the federal gift tax.

14. John Jones buys securities and takes title in the name of "First Bank, as custodian for Sam
Jones, under the [name of state] Uniform Transfers to Minors Act. I hereby direct the bank to
postpone distribution of the securities to Sam until he attains age 35."
The UTMA custodianship will terminate at age ______ because the terms of a UTMA
Custodianship are:

III. CHARITABLE TRUSTS, HONORARY TRUSTS


Four distinctive rules apply to charitable trusts:
#1. Not subject to Rule Against Perpetuities; can be perpetual. (E.g., a trust "to pay the income to
the American Red Cross forever.")
#2. Must be for a charitable purpose. Must confer a substantial amount of social benefit:
education, religion, education, health, relief of poverty, research, governmental purpose.
#3. Must be in favor of a reasonably large number of unidentifiable members of the public at large,
and cannot benefit identifiable individuals. (A trust to pay the income to "my poor relatives" is
not a charitable trust.)
#4. When the stated charitable purpose can no longer be accomplished, may be reformed in judicial
proceeding under the doctrine of cy pres (Old French for "as near as possible").
15. Residuary estate bequeathed to testamentary trustee, "to distribute the income to
organizations, as selected by the trustee, actively engaged in research for the prevention of

Page 40 WILLS AND TRUSTS

LAW SCHOOL LEGENDS

polio." [Note: Trust doesn't have ascertainable beneficiaries. Who can enforce the trust,
then?]

Twenty years later the cure for polio is found, and all research on the disease is comes to an
end. The trustee petitions the court: What should be done with the trust income and
principal? The testator's heirs intervene: Since the purposes of the trust can no longer be
accomplished, the trust should be terminated and the corpus distributed to them. At issue is
whether the court should apply:
General charitable intent that can still be given effect: To devote the entire estate to
medical research, for the prevention and cure of disease.
Specific direction that no longer can be accomplished.
If cy pres is applied, the court will probably instruct the trustee to pay the income to:

16. Turner's will bequeaths his residuary estate "to the Springfield Lighthouse for the Blind."
During Turner's lifetime the Springfield Lighthouse disbands.
________ Can cy pres doctrine be applied to outright gifts to charity as well as to
charitable trusts?
General charitable purpose:

Specific direction:
17. Related doctrine (as to reasoning; applies to private as well as charitable trusts): judicial
modification of trust administrative terms because of changed circumstances. (Most cases
involve restriction on sale of trust assets.)
Leading case: Estate of Joseph Pulitzer. Will created trust for family. The trust
provided that Pulitzer's stock in the New York World newspaper was not to be sold so
long as the trust was in existence. Many years later the World suffered losses to the point
that trustees had to use income from other assets to cover them. Court authorized
deviation under primary intent-specific direction reasoning. Primary purpose of the trust
was to provide for Pulitzer's family; in connection therewith he gave specific direction
that stock should never be sold. But to continue to adhere to specific direction would
frustrate the primary purpose of the trust.
18. Trevor's will creates a trust which provides that his house should never be sold and that his
widow Wanda shall be allowed to occupy the house rent-free for life; upon Wanda's death the
house is to pass to Trevor's son Donald. Some years later the house becomes the center of a
manufacturing district and is no longer suitable as a residence. The trustee petitions for
authority to remove restriction on sale so that house can be sold and proceeds used to buy a
house for Wanda in a more suitable neighborhood.
________ Should a court authorize modification of the trust to remove the restriction
on sale?

LAW SCHOOL LEGENDS

WILLS AND TRUSTS Page 41

Primary purpose of gift:

Specific direction:

19. Client wants to bequeath $25,000 to his sister Sarah as trustee of a 30-year trust, to use the
income [to care for his beloved cat, Figaro] [to pay for a weekly polish-and-wax job on his
beloved Porsche, which is to be parked as a headstone over his grave at the cemetery].
Client's residuary estate will be devised to his brother Bob. Can Client create an enforceable
trust for either purpose?
__________ To have a trust, the trustee must owe enforceable duties to someone. This
it is not a charitable trust, and it has no individual beneficiaries who can
enforce the trust. Cats, nor cars cannot file lawsuits. This type of gift (in
which an animal or an object is the beneficiary) is sometimes called an
honorary trust gift. The trustee is on her honor in deciding whether to
perform the trust. The gift is valid only in the sense that it will be upheld if
Sarah chooses to perform. If she doesn't, the gift fails and there is a
resulting trusthere, in favor of the residuary beneficiary.
Additional problem with such gifts: ____________________________________ unless
the trust is limited to 21 years' duration. Because animals cannot be used as measuring
lives, trust might last longer than LIB + 21 years.

IV. RESULTING AND CONSTRUCTIVE TRUSTS


The term resulting trust is used by the courts in two situations:
(1) Where a trust fails for some reason. E.g., "to pay the income to my best friends."
(2) ______________________________________________________________

20. A pays the purchase price for land, and causes the title to be taken in B's name. A and B are
not related. Thereafter, A brings suit seeking to impress a resulting trust in his favor,
contending that he did not intend to make a gift of beneficial ownership to B, but had some
other reason for taking title in Bs name. Presumption:

Evidence is admissible to show: (1) _________________ or


(2) ____________________________.

Page 42 WILLS AND TRUSTS

LAW SCHOOL LEGENDS

20a. Same facts, except that A is B's father (or sister). Presumption:

Evidence is admissible to show:

Constructive trust IS NOT A TRUST. It is the name given to a flexible equitable remedy designed
to disgorge unjust enrichment. Two elements: (1) wrongful conduct, and (2) unjust enrichment.
21. Tanya had a will that devised all her property to Ann and Bill. Now in her last illness, Tanya
asked a friend to have a lawyer prepare a new will that devised her entire estate to Donna.
Several days later, the friend and the lawyer returned with the new will, which revoked Tanya's
present will. The lawyer began to explain the will's terms to her. Ann and Bill were present;
as soon as they learn what was happening, they create a disturbance and physically prevented
Tanya from signing the new will. Highly agitated, Tanya lapsed into a coma, and died three
days later. She was survived by Ann, Bill, and Donna. Who takes what?
Step 1: Apply the law. Is the new will admissible to probate? No; it was not signed and
witnessed.
Was the old will validly revoked? No; new will with its revocation clause was not
executed.
Law result: Admit the old will to probate. Ann and Bill take Tanya's entire estate
under the will.

Step 2: Apply equity. (1) _________________________________________


plus (2) ______________________________________________________
equals:

22. Fred, a widower, has two children (Sam and Donna) and Sam has two children. After a
heated family argument, Sam bludgeons Fred to death with an axe. Fred left no will. Who
takes Fred's estate? Donna takes , of course. As to the other :

Many states have enacted a "slayer" statute [meaning no constructive trust analysis is
needed]: Killer forfeits interest in victim's estate if he:

LAW SCHOOL LEGENDS

WILLS AND TRUSTS Page 43

Under these statutes, the victim's estate is distributed as though:

22a. Consider the same facts, except that a sizeable number of states have not enacted slayer
statutes. In these jurisdictions:
_________________________________________________________ because we have
____________________________ that would lead to ___________________________.

23. Billy, named as beneficiary on a life insurance policy, was tried and acquitted on charges of
murdering the insured. Nonetheless, in a civil action brought under the states "slayer"
statute, the trial court found as fact that Billy had murdered the insured.
________ Should the decision be affirmed on appeal?

24. Another constructive trust case: In a divorce settlement, Harry agrees to keep a $100,000 life
insurance policy in force that names the couple's child Carl as beneficiary. Harry remarries,
and names his second wife Wendy as beneficiary on the policy. Wendy holds on a
constructive trust for Carl even though Wendy herself was innocent of wrongdoing. She
would be unjustly enriched by Harry's wrongful conduct.
25. Sam promised his sister Marie that he would provide a home for her and her daughter Angie,
if Marie would keep house and serve as hostess in his home. In the presence of witnesses,
Sam handed his brother Hobie Gates a deed naming Hobie as grantee (with no mention of a
trust) saying: "Hold this property in trust until Marie's death, then convey to Angie. This is
in consideration of services rendered by Marie as agreed." Sam died; Hobie denies the
existence of a trust, and claims the home as his own. Marie contacts you regarding legal
action to establish a trust in the property. Discuss issues presented and probable result of
legal action.
a. ________ Valid express trust?
Statute of Frauds: A trust of land must be evidenced by a writing that satisfies
Statute of Frauds; oral trust of land is unenforceable.
However, constructive trust may be imposed if:
(1) Fraud in the inducement. If Hobie orally promised to serve as trustee
but had no intention to perform his promise.
(2) Grantee-trustee was in confidential relationship with grantor-settlor.
(Business associates; father-child; etc.) If Marie can prove Hobie's
promise by clear and convincing evidence, constructive trust imposed:
equity's concern that one would take advantage of a confidential
relationship outweighs the concern of the Statute of Frauds that trusts of
land be in writing.

Page 44 WILLS AND TRUSTS

LAW SCHOOL LEGENDS

b. Suppose that Marie is unable to establish Hobie's promise by clear and convincing
evidence, and no constructive trust is imposed. Would Marie have any action against
Sam's estate?

26. Frank's will devised Greenacre to Jones; the will makes no mention of any trust. Frank's
secretary now alleges that Frank orally told Jones that Jones was to hold the land as trustee
for the secretary's benefit, and that Jones agreed to serve as trustee. Can the secretary enforce
the promise?
YES. Her testimony is admissible to show existence of the promise. If she is able to
establish Jones' promise by clear and convincing evidence, the promise is
enforceable via constructive trust. Equity's concern that one should not be
allowed to be unjustly enriched in violation of his promise outweighs concern that
will beneficiaries should be identified by language in the will. A constructive
trust is imposed to prevent unjust enrichment.
27. Same will, except Frank never told Jones about any trust. With Frank's will is an envelope,
marked "Personal," addressed to Jones. Inside is a typewritten note, signed by Frank,
instructing Jones to hold the land in trust for Frank's secretary. Can the secretary enforce this
as an express trust?
NO

because the will makes no mention of a trust. (And the typewritten note, not
witnessed, is not a valid codicil to the will.)

________ Can the secretary enforce this as a constructive trust?

28. Tim's will devises land "to my good friend Sam Smith, as trustee, for purposes I have already
communicated to him," and devises his residuary estate to Hobie Gates. Smith is willing to
identify Tim's trust purpose and intended beneficiaries, as orally communicated to him by
Tim, and is willing to serve as trustee. Hobie objects, saying no valid trust was created.
What result?
No trust arises. To impose a trust would violate requirement of Wills law that
beneficiaries be named in the will, not by oral testimony. Smith holds on a resulting trust
for Hobie Gates, the residuary beneficiary.

V. CREDITORS' CLAIMS AND SPENDTHRIFT TRUSTS


29. Hobie Gates transfers securities worth $1 million to a bank as trustee: To pay the trust income
to Hobie and his son Aaron in equal shares. On death of the survivor of Hobie and Aaron, the
trust principal is to be distributed to Aaron's children. The trust contains a spendthrift clause
that prohibits voluntary or involuntary transfers of a beneficiary's interest:
"No interest of any beneficiary herein shall be assignable by such beneficiary nor shall it
be subject to the claims of the beneficiary's creditors by garnishment, attachment or other
process."

LAW SCHOOL LEGENDS

WILLS AND TRUSTS Page 45

Aaron borrows $100,000 for a business venture, and signs a note that Hobie signs as
guarantor. In performance of the obligation, Aaron assigns Creditor $10,000 yearly for eight
years out of trust income. Three years later, Aaron instructs the trustee to make no more
payments to Creditor.
a.

Creditor sues Aaron and seeks to enforce the assignment, or to reach Aaron's
interest in the trust by garnishment or execution. Result?

Spendthrift clauses are valid, except as to:


#1. Contracts for necessaries (medical, food, rent)
#2. Child support obligations
#3. Any interest retained by the Settlor
#3A. Revocable trust
#4. Federal tax liens
b.

Does Aaron have an action against the trustee for paying $30,000 to Creditor
under what was clearly an invalid assignment?
________ Beneficiary who participates in breach of trust:

c. Creditor sues Hobie as guarantor and seeks to reach the trust principal or Hobie's
right to of the trust income.
What result as to Hobie's trust income interest? As to any interest retained
by the settlor:

What result as to trust principal? As to interests irrevocably transferred to


third parties:

Exception: Fraudulent conveyance A transfer made with the intent to


defeat, delay, or defraud creditors.
If trust was revocable, Hobie's creditors could reach the entire trust
property, including the principal. If settlor retains the power to revoke
the trust, he is the "owner" of the trust for creditors' rights purposes. In
#29, the trust was silent as to revocability, but in nearly all state an inter
vivos trust is irrevocable and cannot be amended unless the settlor
expressly reserves the power to revoke or amend the trust.

Page 46 WILLS AND TRUSTS

LAW SCHOOL LEGENDS

30. Trust provides: "Income to my daughter Dolly for life, remainder to her children. In addition,
if the trustee determines that the income is insufficient for Dolly's support, the trustee may in
its sole discretion, not subject to challenge by any person, distribute to Dolly so much of the
principal as is needed for her support." Dolly, laid off by her employer, sues trustee to
compel a principal distribution, contending that the income is insufficient for her support
needs.
________ Does Dolly have a cause of action?
Dolly can compel a distribution of the amount she shows is needed for her support.
Purpose of the trust was not to give the trustee discretion; it was to provide for
Dolly's support. It would be an abuse of discretion to not make a distribution if
needed for support.
Conversely, any distribution of principal far in excess of Dolly's actual support needs
(taking into account other resources available to Dolly for her support) is a breach of
trust, and the remaindermen have an action against the trustee.

VI. SELF-DEALING BY FIDUCIARY


31. Trust names friend Hobie Gates as trustee. Hobie borrows $100,000 from the trust, giving
the trust a 6-month note at 9% interest (the then-prevailing bank rate for loans.) Hobie uses
the borrowed funds to buy a rental property for $100,000, and six months later sells the
property to Hazel for $200,000. He repays the $100,000 (plus interest) to the trust. What are
trust beneficiaries' rights . . .
Against Hobie Gates?

a. Trustee cannot buy or sell trust assets to himself.


b. Trustee cannot borrow trust funds.
c. Trustee cannot loan funds to the trust, and any interest earned on such a loan must be returned
to the trust. Any security interest received in connection with such a loan is invalid.
Self-dealing rules can be waived by the settlorbut the ability to waive the self-dealing rules
is rarely tested.
d. Trustee cannot profit from serving as trustee (except for being compensated), as by taking
advantage of confidential information received in his capacity as trustee, or by receiving a
commission for the sale of trust property.
e. Corporate trustee cannot buy its own stock as a trust investment. (But it can retain its own
stock if a part of the estate received by it as trusteeprovided that retention of the stock is
prudent.

LAW SCHOOL LEGENDS

f.

WILLS AND TRUSTS Page 47

Duty to segregate trust assets from personal assets, and duty to earmark trust assets by taking
title in the trustees name. Trustee cannot commingle trust funds with her own. If commingled
funds used to purchase asset and the asset goes down in value, conclusive presumption that (to
the extent available) personal funds were used. If the asset goes up in value, conclusive
presumption that (to the extent available) trusts funds were used.

Statute of Limitations does not begin to run on action against a fiduciary (trustee, executor,
guardian) unless and until he
(1) repudiates the trust [buzz words used by the courts] by denying existence of trust as to
the particular assets in issue;
(2) dies or resigns; or
(3) gives accounting that makes full disclosure of the facts on which action would be based
(e.g., an accounting that shows, not just that the trust loaned funds to someone, but that
the trustee was the borrower).
Whenever a trustee breaches any fiduciary duty (commits a "breach of trust"), whether self-dealing,
speculative investment, exercises a power not given to the trustee) in addition to bringing action to
remove trustee, beneficiary has option:
(i) He can ratify the transaction and waive the breach. (E.g., if speculative mining stock
goes up in value, "Thanks for doing such a good job of investing.")
(ii) He can sue for resulting loss. Name of the action is surcharge. Moreover, if (as in #31)
the case involves self-dealing, under the no further inquiry rule breach of a fiduciary
duty is an automatic wrong; good faith reasonableness, is no defense. Only issue in a
self-dealing case: measure of damages.
If (as in #31) trustee borrows trust funds and invests the proceeds, if value of purchased property
goes up in value, beneficiary can "trace" and claim the property for the trust via the imposition of a
constructive trust.
31a. Do the beneficiaries have any action against Hazel, who purchased the rental property from
Hobie, when she knew that Hobie was a trustee? (The rental property is now worth
$300,000)?

31b. Do the beneficiaries have a remedy if Hazel was Hobie's sister?


____________ Self-dealing rules also apply to loans or sales to a relative, and to a
business entity of which the trustee is an officer, partner, employee or
principal shareholder (sometimes called "indirect self-dealing").

VII. TRUST ADMINISTRATION PROBLEMS


B.

INVESTMENTS
32. Trust (current value $800,000) provides for income to Wife for life, remainder to Children.
The trust does not authorize distributions of principal to Wife. Current investments include
6% corporate bonds ($200,000) yielding $12,000 in annual income, and common stocks
listed on New York Stock Exchange ($600,000) producing cash dividends at an average of

Page 48 WILLS AND TRUSTS

LAW SCHOOL LEGENDS

2% ($12,000 per year). Trustee sells some common stock and invests the $200,000 sale
proceeds to buy stock in an IPO ("initial public offering") for Bodacious Inc., a three-yearold Internet start-up company that has lost money every year, has never paid dividends, and
is not likely to do so for at least five years. Six months later, the Bodacious stock has
declined in value to $80,000. Should the trustee be held liable for the $120,000 loss?
Former (pre-1990) law in most states: probably YES. (i) investing in a company with no
investment history or track record too speculative. (ii) investing 25% of trust in asset that
produces no income violated duty of fairness trustee owed to the income beneficiary vis a
vis the remaindmen.
Current law: Nearly all states have enacted the Uniform Prudent Investor Act ["UPIA"],
which is based on the modern portfolio theory of investing.
Trustee must establish a custom-tailored investment strategy for each trust, taking
into account such factors as:
-- general economic conditions,
-- the possible effect of inflation or deflation,
-- the expected tax consequences of investment decisions or strategies,
-- the role that each investment plays within the overall trust portfolio,
-- the expected TOTAL RETURN from income and capital gain,
-- needs for liquidity,
-- an asset's special relationship or value to the purposes of the trust or a
beneficiary, and
-- any differing interests of the income beneficiaries and the remaindermen.
________ But doesn't the investment's sharp decline in value create a res ipsa case that
the investment was imprudent, leading to trustee liability?

But what of the fact that the trustee knew that the IPO stock was unlikely to pay
dividends; and that for a $800,000 (oops! now $680,000) trust portfolio, the ordinary
income (bond interest and cash dividends) to be distributed to Wife will only be about
$20,000?
#1: Under UPIA, investment returns are measured by:

#2: Under the Uniform Principal & Income Act, trustee can exercise:

Starting point: Trustee distributes "income" items (interest income, rental income,
dividends on common stocks, etc.) to the income beneficiary; and adds capital gains (part
of proceeds of sale of a principal asset) to the corpus of the trust. However, .
Factors to be considered in exercising adjustment power (i.e., power to adjust total
return between income and principal, and allocate capital gain to principal):

LAW SCHOOL LEGENDS

WILLS AND TRUSTS Page 49

-- purpose and expected duration of the trust,


-- intent of the settlor as to respective interests of the beneficiaries,
-- the net amount of ordinary income and capital gain available for allocation,
-- circumstances of the beneficiaries,
-- the need for liquidity, regularity of income, and preservation and appreciation
of capital,
-- any increase or decrease in value of the trust assets,
-- whether the trust gives the trustee a power to distribute principal,
-- effect of economic conditions and effects of inflation and deflation, and
-- anticipated tax consequences of an adjustment.
________ Can a settlor authorize trustee to make speculative investments that are not
within the "prudent investor" standard?
All provisions of a particular states Trust Code are "default" rules. They apply absent
contrary provision by the settlor. A settlor can "write her own rules," and modify the
otherwise applicable statutory provisions.
________ In a state that has enacted the Uniform Trustee Powers Act, can a trustee give
a 99-year lease without court approval?
________ .... Can trustee enter partnership to drill for oil without court approval?
Uniform Trustee Powers Act (paraphrased):

Model answer: "The [name of state] Trust Code, which applies to all trusts in
[state] except to the extent the trustee's powers are expanded or limited by the
settlor, gives broad fiduciary powers. Specifically, the Trust Code expressly
authorizes a trustee to [do whatever the question involves]."
Exception: There are two situations where the answer is "NO, the trustee
DOESN'T have the power to do this." (There is a third casewhere the settlor
limits the trustee's powersbut that one isn't tested upon.)
1. ___________________________________________________
2. ___________________________________________________
33. Tom's will created a trust that named Larry, Curly and Moe as co-trustees. Some years later,
Larry and Curly want to sell Greenacre (a trust asset) and invest the sale proceeds in other
assets. Moe objects, and takes the position that since the trustees are not unanimous,
Greenacre cannot be sold. The trust instrument is silent on the question. Under the Trust
Codes of most states, is Moe correct?

________ (majority rule): If there are two or more trustees ____________________

Page 50 WILLS AND TRUSTS

LAW SCHOOL LEGENDS

(Absent contrary provision)

In a few states (e.g., Connecticut), actions of co-trustees must be ________________.

VIII. EARLY TERMINATION OF TRUSTS


Beneficiaries can terminate a trust prior to the time fixed for its termination if:
(i)

ALL beneficiaries, all of whom are sui juris (over age 18, all have capacity), consent; and

(ii)

there is no further trust purpose of the settlor to be served.

There is no requirement that the trustee must consent to an early termination.


Why is it that trusts can hardly ever be terminated early under the above rule?

Spendthrift clause makes the trust indestructible (beneficiaries cannot terminate without settlor's
consent). Idea is that there is a further trust purpose of the settlor to be served: To provide
beneficiary with income interest that could not be assigned by him or reached by his creditors. To
terminate the trust would defeat a material trust purpose of the settlor.
Settlor can terminate a trust (including a spendthrift trust) even though it is irrevocable if all
beneficiaries consent. Point: If all parties who are interested in the trust say let's quit, no point in
continuing. However, that means ALL beneficiaries: They must be sui juris (i.e., can't be any minor
beneficiaries).
34. Mom died 15 years ago, leaving a will that created a trust: "Income to my daughter Mary
until she attains the age of 35, at which time the trustee shall distribute the trust principal to
her." Mary, who just turned 26, petitions to terminate the trust, pointing out that she is a CPA
and has a MBA from the Harvard Business School, has been successful in several business
ventures, and doesn't need a trust to manage the assets.
________ Should the court order termination of the trust?
What was the primary purpose of the trust?

IX. POWERS OF APPOINTMENT


35. Tom died in 1990, leaving a will that bequeathed property in trust: "to pay the income to my
daughter Dana for life, and on her death to distribute the trust principal to such persons,

LAW SCHOOL LEGENDS

WILLS AND TRUSTS Page 51

including Dana's estate, as she appoints by her last will. If Dana does not exercise this power
of appointment, on Dana's death the trustee shall distribute the trust principal to Dana's
children." [Purpose of a power of appointment: permits the income beneficiary to designate
the remaindermen.]
Tom was the donor of the power of appointment as his will created the power.
Dana is the donee of a ______________________________________ testamentary
power of appointment, because she is not limited in the class of beneficiaries to
whom she can appoint; she can appoint the property to herself, her creditors, OR her
estate.
Dana's children are takers in default of appointment, as they will take the property
on Dana's death if the power of appointment is not exercised.

35a. Dana died three weeks ago, leaving a will that bequeaths her residuary estate "one-half to
my husband Harry and one-half to my son Steve."
_______ [Majority rule] Did the residuary clause in Dana's will operate to exercise the
testamentary power of appointment, even though her will made no reference
to the power of appointment?

Who takes the trust property on Dana's death, then?

36. Mom's will creates a trust: "Income to my daughter Beulah for life, and on her death
principal to such of Beulah's descendants as she shall appoint by her last will. In default of
appointment, to Beulah's children in equal shares."

Beulah has been given a life estate and a ________________________ testamentary


power of appointment because she is limited in the class of persons to whom she can
appoint. She cannot appoint to herself, her estate, or her creditors.

36a. Beulah dies ten years later. Her will devises "all my property, including any property over
which I may have a power of appointment, to my daughter Diane."

________ Did Beulah's will exercise the special testamentary power in favor of
Diane?

36b. Same facts, except that the trust provided: ... and on Beulahs death principal to such
of Beulahs descendants as she appoints by a will that specifically refers to this power
of appointment. In default of appointment, to Beulahs descendants. Beulah dies ten
years later. Her will devises "all my property, including any property over which I may
have a power of appointment, to my daughter Diane."

________ Did Beulah's will exercise the special testamentary power in favor of
Diane?

Page 52 WILLS AND TRUSTS

LAW SCHOOL LEGENDS

36c. Original facts (no "specific reference" clause). During Beulah's lifetime, for a
consideration she enters into a contract under which she agrees to execute a will that
exercises the power of appointment in favor of her son Roger. Beulah later dies
leaving a will that exercises the power in favor of Roger. Valid exercise of the power
of appointment?
. . . because:

(Also, to hold otherwise would effectively transform Beulahs testamentary


power of appointment into an inter vivos power; and Beulah was not
supposed to exercise the power during her lifetime, but only by will.)

Вам также может понравиться